SlideShare ist ein Scribd-Unternehmen logo
1 von 50
P a g e | 1
“A liberal secular legal system which excludes religious influence in the
public sphere would adequately respond to the challenge of achieving
equality…”
In view of secular liberalism’s understanding of equality and its critique, how
accurate is this statement?
Brandon Toh Kah Meng
April 2016
Dissertation submitted in partial fulfilment for the degree of
LLB. Law (Hons)
Law
University of Hertfordshire
P a g e | 2
Abstract
The rise of religious pluralism has triggered much debate and an ambiguous academic
response vis-à-vis the best approach toward achieving equality. This dissertation addresses
an ambitious research subject area: Does religion have a place in a secular public sphere
and what are the effects of its exclusion and inclusion on achieving equality in a religiously
diverse society.
This problem is tackled by scrutinizing the two different ends of the spectrum of secularism –
“Aggressive”and “Open” Secularism.Aggressive secularism will be discussed in light of
Laïcité and the Burqa Ban Case1
under the category of “Neutrality-by-Exclusion”while Open
secularism will be studied in light of the Canadian legal system and the seminal case of
Multani2
under the category of “Equality-by-Accommodation”.
By analysing both positions, pointing out conceptual flaws and debunking misconceptions,
the dissertation establishes that an exclusion of religious influence from the public sphere is
the best way to achieve equality.
1 S.A.S. v. France [GC], App. No. 43835/11,2014 Eur. Ct. H.R.
2 Multani v. Commission scolaire Marguerite-Bourgeoys [2006] 1 S.C.R. 256,
P a g e | 3
Acknowledgements
I would like to extend my sincere and heartfelt gratitude towards all the people who have
helped and supported me in this ambitious research project. Without their active guidance,
help, cooperation and encouragement. I would not have made headway in the project.
I am especially thankful to Dr. Camille Pommel for her conscientious guidance and
encouragement. The complexity of my research subject area required extended periods of
consultation which she graciously accommodated to. I would also like to extend thanks to Dr.
Felipe Romero Moreno,the dissertation course manager, for promptly replying enquiries and
for the facilitation of the program. The University of Hertfordshire has provided me with not
only this opportunity to challenge my literary capabilities but also a holistic legal education
which contributed to my academic capacity – for that I am forever indebted.
Lastly, I would also like to acknowledge with a deep sense of reverence, my gratitude towards
my mother who has always been there for me and I hope this dissertation will do her proud.
SincerelyThanking You
Brandon Toh
P a g e | 4
Table of Contents
Abstract .....................................................................................................................2
Acknowledgements ...................................................................................................3
Table of Contents ......................................................................................................4
1 Introduction...........................................................................................................5
2 Is There A Conflict?: Religion vs. Law and Equality ......................................6-10
3 History of Liberalism...........................................................................................10
3.1 Political Foundations: Birth of Classical Liberalism ...............................10-11
3.2 Liberalism and Democracy..................12-1Error! Bookmark not defined.
3.2.1 Principles of the Liberal Solution... 1Error! Bookmark not defined.
3.3 Liberalism and Utilitarianism.................... 1Error! Bookmark not defined.
3.4 Liberalism and Equality ..........................................................................14-15
3.4.1 Classification of Liberalism.........................................................15-19
4 Two Prongs of Liberalism:"Neutrality-by-Exclusion" vs "Equality-by-Accommodation"
19-20
4.1 "Neutrality-by-Exclusion"........................................................................20-22
4.1.1 “Aggressive Secularism“- Laicite.....................................................22
4.1.2 The Burqa Ban Controversy.....2Error! Bookmark not defined.-27
4.1.3 Secularism vs. Secularization.................................................27-28
4.1.4 Analysis: Secularism is a Stronger Ideal than Perceived ..........28-30
4.2 "Equality-by-Accommodation"................... Error! Bookmark not defined.1
4.2.1 “Open Secularism” - Canada....Error! Bookmark not defined.1-33
4.2.2 The Kirpan Controversy............Error! Bookmark not defined.3-38
4.2.3 Analysis: Conceptual Flaws of the “Accommodative” ApproachError!
Bookmark not defined.8-41
5 Conclusion.....................................................................................................42-44
Biblography.........................................................................................................45-50
P a g e | 5
1. Introduction
The rise of religious pluralism has triggered much debate and an ambiguous academic
response vis-à-vis the best approach toward achieving equality. This dissertation addresses
an ambitious research subject area: Does religion have a place in a secular public sphere
and what are the effects of its exclusion and inclusion on achieving equality in a religiously
diverse society.
On one side, there is the ‘aggressive’ secular position – often associated with the French
Laïcité. This position advocates the strict separation of public and private spheres, relegating
religion to the latter. On the other hand, there is the ‘open’ secular position – often
associated with the Canadian legal and political system. This position prioritises the
accommodation of religious belief and freedoms even if it is at the expense of the principle of
neutrality. These juxtaposing positions are the manifestation of legal and political struggles
to grapple with a jurisdiction’s constitutional identity and foundational values: ‘rule of law,
democracy, human rights, and, last but not least secularism.’3
In our age of increasing religious pluralism, the best way to manage diversity has once again
come into question. Equality, the right of different groups of people to have a similar social
position and receive the same treatment, has become a complex puzzle that this dissertation
will attempt to solve. This will be done by analysing the two secular positions represented
above – “Aggressive” and “Open” secularism. The dissertation will first elucidate the
problematic relationship between religion and equality. Secondly, study of the development
liberalism will first be done to give historical context and an understanding of the underlying
principles of these two variations of secularism. Aggressive secularism will be discussed in
light of Laïcité and the Burqa Ban Case4
under the category of “Neutrality-by-Exclusion”
3 L. Zucca,‘A Secular Europe: Law and Religion in the European Constitutional Landscape’ (1st edition, Oxford
University Press 2012) at1
4 S.A.S. v. France [GC], App. No. 43835/11,2014 Eur. Ct. H.R.
P a g e | 6
while Open secularism will be studied in light of the Canadian legal system and the seminal
case of Multani5
under the category of “Equality-by-Accommodation”. A critical analysis of
the critique of these two positions will entail. Following which, in the conclusion, the
dissertation will re-assert the stance that aggressive secularism which excludes religion from
the public sphere will most appropriately respond to the challenge of achieving equality.
2. Is there a Conflict?: Religion vs. Law and Equality
Religion is a complex, historically and socio-culturally embedded concept. In this
section, the dissertation will explore the fundamental reasons why a conflict between law
and religion exists. The two main characteristics that make it a formidable obstacle in
achieving equality are:
Firstly, there is a plethoric variety of religious ideology. Secondly, there is increasing
acknowledgment that observations, or claims, of religions are inevitably rooted in competing
religious and cultural traditions themselves.6
This seems to exclude any possibility of
transcultural or universalistic definitions or second-order observations7
of religions.
The spectrum of types of religion reach far beyond the main Abrahamic-beliefs and
these contemporaneous types of religions differ in many dimensions:
(1) Is the belief centred upon a distinguishability between an immanent and a
transcendent - world?8
And if so, which semantics9
?
(2) Are they belief-centred or centred on practices?
5 Multani v. Commission scolaire Marguerite-Bourgeoys [2006] 1 S.C.R. 256,
6 Habermas, J. ‘Notes on a Post-secular Society’ (2008), at 4.
7 “The first-order observer lives in a world that seems both probableand true. By contrast,the second-order
observer notices the improbability of first-order observation.”Luhmann, N. ‘Observations on Modernity’ (1st
edn, Stanford University Press 1998)
8 Transcendence refers to the aspect of a god's nature and power which is wholly independent of the material
universe, beyond all physical laws.This iscontrasted with Immanence, where a god is said to be fully present in
the physical world and thus accessibleto creatures in various ways.
9 The study of meaning. It focuses on the relationship between signifiers—likewords,phrases,signs,and
symbols—and whatthey stand for, their denotation.
P a g e | 7
(3) Is the religious belief transmitted orally (e.g. as mythos) or by written text.
(4) Is there a propensity for experts (e.g. rabbis, theologians or sheiks etc) to make their
holy scripture consistent and systemic (dogmatising canons, rules and regulations)
(5) Is the religious community bound solely by shared practices or beliefs, or does it
revolve around a more formalised and hierarchically structured organisation (orders,
temples, mosques, churches, congregations) and authoritative rulers (bishops,
popes, patriachs)?
The corollary of which is a complication in any goal in achieving equality in an increasingly
pluralistic world.
The recent conditions of increasing religious pluralism seems to be accompanied by
a fragmentation of organised religions which gradually insist more vigorously on respect for
their belief and on living according to the commandments of their individual creeds in public
spheres. The result of which is pressure exerted onto existing forms of institutionalisation of
religions and their management by the state. These developments have prompted a
reconsideration of the traditional conception of religion in social sciences as well as
normative disciplines, especially law and jurisprudence. The essay will focus on the latter
and discuss the practical challenges it brings.
Religiosity has in every legal jurisdiction to different degrees made its mark. The US
legal jurisdiction has been met with famous religious cases concerning polygamy, the flag
salute, conscientious objections and other exemptions as well as the use of drugs. The
Supreme Court’s mounting difficulties in finding defensible definitions of religion under
conditions of greater and manifestly visible religious diversity are excellently analysed by
Galanter and in a HLR Note.10
10 Galanter,M. ‘Religious Freedoms in the United States: A Turning Point’, (1966) Wisconsin LawReview 2, at
235 and 260; Note ‘Developments in the Law: Religion and the State’, (1987)100 Harvard LawReview 7, at
1622-1631,1647.On similar difficulties in other countries and international covenants,see V. Bader,
Secularism or Democracy? Associational Governance of Religious Diversity (2007),at 301 (note 3).
P a g e | 8
In most western jurisdictions, where bias was originally in favour of the national
majority religion of Christianity (protestant), the prospective of the majority religion, implicitly
or explicitly endorsed by the Court, has been disguised as an objective standard. This is
perceptible in Reynolds v. United States11
where a federal law banning polygamy was
upheld, undermining the Mormon religion. In Braunfeld v. Brown12
, Pennsylvania courts
which required stores to be closed on Sundays, undoubtedly for the benefit of Christian
religious practices, held that it was not an unconstitutional interference with religion as
described in the first amendment of the United States Constitution even though the Jewish
appellants submitted that it unduly burdened them since their religion required them to close
their stores on Saturdays as well.
European societies too face a growing number of conflicts due to increasingly
concerning claims based on freedom of religion. The reason however, is neither a revival of
archaic inner-Christian controversies nor a new hostility of the state vis-à-vis religion or a
new identification with a single denomination as was the tradition in many European
countries. Rather, the conflicts have their source in the growing multiculturalism of European
societies, caused by the immigration of members of non-Christian beliefs.13
These beliefs
often, alike Christian denominations, lay claim to a divine absolute truth, but unlike Christian
denominations, have not undergone the process of historicization and contextualization of
these divine revelations that permits the Christian churches to relatively greater extents
adopt a more distant approach on the imposition of their values and commandments.
Moreover, some religious communities adhere to religious- influenced laws which are
blatantly incompatible with our modern conception of human rights. For example, within
Sharia law, there are specific forms of punishment for certain crimes. Strict governance
11 98 U.S. (8 Otto.) 145 (1878)
12 366 U.S. 599 (1961
13 D. Grimm, 'General Laws and Religious Norms' [2009] 30(6) Conflicts Between General Laws and Religious
Norms 2370
P a g e | 9
under Sharia law dictates for example, that fornication is punishable by stoning, the
consumption of alcohol by lashing, and theft by the amputation of limbs.
The argument for religious autonomy and against religious coercion can be
categorized in two groups: inter-group relationships (one group imposing its views on
another), and intra-group relationships (a group imposing its views on its own members).
The crux lie in that no one group should have the right to coerce the entire society into
abiding by its conception of what the law should be based on their religious ideology. For
example, some immigrants hide behind the guise of ‘multiculturalism’ as a pretext for
imposing patriarchal practices on women or children. This would include stifling the rights of
women or even traditional customs such as coercive arranged marriages of sorts.14
This dissertation proposes that the best method in achieving equality is to establish a
liberal secular legal system which excludes religious influence in the public sphere because
the accommodation of religion fundamentally contradicts the concept of equality.
In order to grasp this notion, let us consider a hypothetical scenario where religious
minorities are left alone to run their own communities in accordance with their religious
commandments. This would entail accepting any contravention of human rights. For
instance, parliament would have to ‘respect’ a minority’s illiberal consensus with regard to a
woman’s role in society, limiting them from education or holding public office. Kymlicka and
Cohen-Almagor argue that if women who dislike this restriction can easily leave the
community and enter the larger society, and if the minority group has some historical claim
to local self-government, then this may mean that it would be wrong for us to coercively
interfere and prohibit that practice. 15
In spite of that, the fact is, wrongful intervention justifies
not the moral legitimacy of the practice.
14 For a discussion of conflicts between some EastAsian immigrantgroups and the British government over
girls’education and arranged marriages,see Poulter (1987), 589-615;Barber (2004), and Ghimire (2006).
15 Kymlicka,Will, ‘Multicultural Citizenship: A Liberal Theory of Minority Rights’, (Oxford: Oxford University
Press 1995).and Raphael Cohen-Almagor, “Ethnocultural Minorities in Liberal Democracies”, in: R. Cohen-
P a g e | 10
Can a liberal secular society in the name of equality then exclude the religious belief
of the majority from the public sphere, hence undermining the cultural history of the
jurisdiction, and the minority, whose members intend to integrate into the community? This
dissertation will explore that by first scrutinising the historical context and the synergy of
liberal principles that lead to our modern conception of liberal secularism.
3. History of Liberalism
3.1. Political Foundations: Birth of Classical Liberalism
Liberal ideas seem to have only surfaced in European politics in the early 16th
century, however, liberalism can be traced back to the Middle Ages. In the Middle Ages, the
rights and responsibilities of an individual were decided on a hierarchical social system. This
feudal stratification of society in the Middle Ages gradually dissolved due to the impact of the
slow commercialization and urbanization of Europe at that time, the intellectual ferment of
the Renaissance, and the spread of Protestantism in the 16th
century. This resulted in a
convoluted mess of fear, instability and chaos within the region. The response to this civil
dissension at that time, was monarchical absolutism.
By the end of the 16th
century, the authority of the papacy had been broken in most
of northern Europe, and the rulers of different regions resorted to consolidating unity of his
realm by enforcing conformity either to Roman Catholicism or to the ruler’s preferred version
of the Christian faith (such as Protestantism). The imposing of these juxtaposing schools of
belief, albeit of the same religion, onto the governance of these societies culminated in the
Thirty Years’ War (1618 – 48). The immense damage to the European region, alongside the
fact that no creed managed to eradicate its enemies, led to a general consensus that
toleration was the lesser of two evils and that neglecting different beliefs of citizens was
inimical to prosperity and good order.
Almagor (ed.), Challenges to Democracy: Essays in Honour and Memory of Isaiah Berlin, London: Ashgate
PublishingLtd., 2000, 89-118.
P a g e | 11
Post 16th
century, different industries and commerce expanded and economic
policies based on mercantilism, which was an advocacy of government intervention in a
country’s economic affairs to increase wealth, were adopted. However, this intervention
increasingly served established interest and inhibited enterprise. The newly emerging middle
class challenged this. This challenge was a significant factor in the great revolutions that
rocked England and France in the 17th and 18th centuries—most notably the English Civil
Wars (1642–51), the Glorious Revolution (1688), the American Revolution (1775–83), and
the French Revolution (1789). Classical liberalism as an articulated creed is a result of those
great collisions.
Toleration emerged in the seventeenth century and was depicted as the best
response to these religious conflicts. It was recognised a key political virtue which the state
imposed as a legal obligation. An example of its legal implementation is the Act of Toleration
168916
. These political ideals were given formal expression n philosophers like John Locke.
He regarded toleration as an imposition of reason and the lack thereof is explained in terms
of being carried away by ‘irregular positions.’ Both the Act of Toleration and Locke’s ‘Letter
Concerning Toleration’17
are examples of a moralizing attitude of the political and intellectual
elite of that time. The ability of regard differing beliefs as conditionally acceptable and
imposing reason from an ideal moral viewpoint formed the foundations of most early liberal
theories. These normative theories rely on the assumption about the fallacy of some
religious beliefs and the rightness of some liberal values. These theories form the bedrock of
the conception of modern secularism.
16The subtitlesays:‘An Act for exempting their Majesty’s Protestant Subjects dissentingfrom the Church of
England from the Penalties of certain Lawes.’
17 Locke, J., ‘A Letter Concerning Toleration and Other Writings'. (Indianapolis:Liberty Fund, 2012).
P a g e | 12
3.2. Liberalism and Democracy
In the eighteenth to the early party of the nineteenth century, a political program was
formulated from these ideals to serve as a guide to social policy. Early liberals strived to free
individuals from religious conformity and aristocratic privilege- social constraints that were
preserved by the powers of government. Their aim was to limit the power of government
over individual rights but hold them accountable to the governed individual. However,
nowhere was this program ever completely carried out.
Classical liberalism fell victim to ambivalence, torn between the emancipating
tendencies that entailed the revolutions and fears that a universal “majority rules” franchise
would undermine private property. John Adams in his Defense of the Constitutions of
Government of the United States of America18
accentuated the dangers of allowing the
majority to control all branches of government, he declared: “debts would be abolished first;
taxes laid heavy on the rich, and not at all on others; and at last a downright equal division of
everything be demanded and voted.”
Most eighteenth and nineteenth century liberal politicians feared popular sovereignty
and for a long period limited suffrage to property owners. In Britain, even the Reform Bill of
186719
held onto property qualifications for the right to vote. In France, only 200,000 out of
approximately thirty million citizens were qualified voters despite the ideal of universal male
suffrage proclaimed in 1789 and reaffirmed in the Revolutions of 1830. In the United States,
it was only until 1860 that universal male suffrage prevailed albeit only for white citizens.
Racial and sexual prejudice also served to limit the liberal franchise and in the United States,
slavery deprived large numbers of people of freedom.
18 John Adams, A Defence of the Constitutions of Government of the United States of America (3rd edn, The
Lawbook Exchange Ltd, 1787)
19 30 & 31 Vict. c. 102
P a g e | 13
Liberals had to reconcile the ‘majority rules’ principle with the power of majority to be
limited. The aspiration of accomplishing this in a manner consistent with democratic
principles lead to the advent of a three-part liberal solution.
3.2.1. Principles of the Liberal Solution
The first was the conception of the separation of powers. This was an adaptation of
the trias politica principle of the Greeks which stipulated the distribution of powers between
functionally differentiated agencies of government as the legislature, the executive and the
judiciary. This formed as a system of check and balance so no single limb of the government
held all the power.
The next step of development was the implementation of periodic elections. This was
a system that allowed for democratic decisions of the majority distributed over time. It
pressures parliament to seek popular mandate and act on the voices of the majority that
elected it. This expression of liberalism was the first step in the formation of the constitutional
democracy which we know today – the checking of the current majority’s influence on
decisions over the verdicts of majorities that precede it.
The third part of the solution followed from liberalism’s commitment to the freedom
and integrity of individuals. Liberals believe that even though a citizen shares a social
contract with the community, they as individuals have rights which the state may not
encroach upon. In order to properly implement a democratic model of governance,
individuals must have rights such as freedom of expression, freedom to associate and
organize and freedom from fear of reprisal. In liberal democracies, individuals should also
maintain the private rights which concern not the state. These rights range from the practice
of religion to the raising of children by their parents. Affirmations of these rights were
declared in legal documents such as the British Bill of Rights (1689), the U.S. Declaration of
Independence (1776) and Constitution (ratified 1788), the French Declaration of the Rights
of Man and of the Citizen (1789). These documents accentuate the liberal perspective that
P a g e | 14
freedom encompasses not only the occasional right to vote but fundamental rights of people
to live their own private lives.
It was the liberal insistence that “all men are created equal” that lead to a slow but
steady expansion of the liberalism franchise which eventually led to the development of the
liber secular model this dissertation is focused on.
3.3. Liberalism and Utilitarianism
The next stage of development for Liberalism is the influence of utilitarianism. In the
late eighteenth century, Jeremy Bentham20
and his student John Stuart Mill21
invoked a
doctrine of utilitarianism- “it is the greatest happiness of the greatest number that is the
measure of right and wrong.”22
This became a major element in the liberal conception of
state policy objectives. This concept was inspired by the notion of a market economy.
Utilitarians advocated a political system that would guarantee its citizens the maximum
degree of individual freedom of choice and action consistent with efficient government and
the preservation of social harmony. In John Stuart Mill’s On Liberty23
, he propounds on
utilitarian grounds that the state may regulate individual behaviour only in cases where the
interests of others would be perceptibly harmed. It is clear the influence utilitarianism has
had on the philosophical foundations of political liberalism. Its classic advocacy of individual
liberties have played an integral role in the modern liberalism model we understand today. In
addition, the Utilitarian maxim of ensuring that the liberties afforded to the community is
“consistent with efficient government and the preservation of social harmony” forms the
basis of the understanding of the liberal secular model we know today.
20 Jeremy Bentham, (born February 15, 1748,London, England—died June 6, 1832, London), English
philosopher,economist,and theoretical jurist,the earliestand chief expounder of utilitarianism.
21 John Stuart Mill,(born May 20, 1806,London, Eng.—died May 8, 1873, Avignon, France), English
philosopher,economist,and exponent of Utilitarianism
22 Bentham, J., ‘A Fragment on Government.’ (London,1776), Preface (2nd para.).
23 Mill,John Stuart. ‘On Liberty’. (London: Longman, Roberts & Green, 1869);
P a g e | 15
3.4. Liberalism and Equality
3.4.1. Classification of Liberalism
There is a common misconception that liberalism places the interests of a part of society
– the propertied classes, capitalists, the entrepreneurs- above the interests of other
classes.24
This assertion is mistaken. Liberalism has always placed emphasis on the welfare
of the whole, not that of any special group. Basil Mitchell identifies three kinds of liberalism
(1) “A liberalism which bases itself on a conviction about the nature of man and of his
destiny and which insists on the need of the individual for freedom to choose the way
he shall live. It is a liberalism which has an explicit metaphysical context.
(2) A liberalism which is sceptical of religious and metaphysical claims and which sees
as the supreme values of human life the discovery of truth and the attainment of
happiness. It regards the need for individual freedom partly as a self-evident truth,
partly as a pre-condition of the discovery of truth and the attainment of happiness.
(3) A liberalism which is sceptical not only of religious and metaphysical claims but of
morality also, except in so far as it is a necessary condition of the survival of any
society. It sets a high value on human beings and on the ideal projects men desire
for themselves, although it does not regard these ideals as, in any sense,
objective.”25
This summarises well the different variations of liberalism we know today. The first
two kinds of liberalism presents a type of theory about the ‘good life’. The good life is an
24 Ludwig Von Mises,Liberalism (1 edn, The Foundation for Economic Education,Inc Irvington-on-Hudson,New
York 10533 and Cobden Press 2002) 7
25 Basil Mitchell, ‘Law, Morality, and Religion in a Secular Society’ (Oxford: Oxford University Press,1967),p.
98.
P a g e | 16
autonomous life, either because of some religious or metaphysical understanding, or
because the good life involves ‘the discovery of the truth and the attainment of happiness’.26
The third kind of liberalism is different. While it lacks clarity on whether its scepticism
denies the existence of moral truth claims, or it denies their relevance in public life, it is a
liberalism that each human being chooses without any discrimination between ideals or
projects.27
Joseph Raz claims that:
“It (Liberalism) is identified by a series of political causes espoused by liberals over the
centuries, by a variety of claims about the working of society and the economy, and by a
cluster of ideas concerning the fundamental principles of political morality. … The specific
contribution of the liberal tradition to political morality has been its insistence on the respect
due to individual liberty.”28
It is in determining what this respect requires that have led to differing variations of the liberal
political concept. For example, Dworkin propounds that the key value is ‘equality’ and the
requirement that all citizens are treated as equals. He then asks what this requires in terms
of political morality (the ethics, or ethical standards, of public or political life.):
“What does it mean for the government to treat its citizens as equals? That is, I think, the
same question as the question of what it means for the government to treat all its citizens as
free, or as independent, or with equal dignity…It may be answered in two fundamentally
different ways.
26 ibid.
27 Peter Griffith,'Christianity, liberalism and the separation of law and morals' [2001] 1(1) UCL Jurisprudence
Review
28 Joseph Raz, ‘The Morality of Law’ (Oxford: Clarendon Press,1989),p. 1-2.
P a g e | 17
(1) The first supposes that the government must be neutral on what might be called the
question of the good life.
(2) The second supposes that government cannot be neutral on that question, because
it cannot treat its citizens as equal human beings without a theory of what human
beings ought to be.”29
The first answer requires the neutrality of a government and if it prefers, in any
decision or policy one conception of the good life over another, it cannot treat its citizens as
equals. The second argues that governance requires an unbiased conception of what it is
that makes life good, so as to treat each citizen in a way a good or wise person would
expect. Dworkin is clear that in his view, ‘liberalism takes, as its constitutive political
morality, the first conception of equality.’30
With this view, Dworkin agrees with John Rawls, who draws the distinction between
views, such as those of Plato, Aristotle and the Christian tradition as represented by
Augustine and Aquinus, which hold that there is only one conception of the good ‘to be
recognised by all citizens who are reasonable and rational’31
, and the view that ‘there are
many conflicting reasonable comprehensive doctrines with their conceptions of the good,
each compatible with the full rationality of human persons, so far as that can be ascertained
with the resources of a political conception of justice.’32
This aptly describes the complex relationship between liberalism and secularism. The
development of liberalism lay the groundwork for the birth of secular theories. These
theories, at least at their conception, were based on the benevolence of religion, reflecting
magnanimous and optimistic assumptions that the nature of religion was cooperative. Where
one can assume that religions are sensible and responsive, ready to dialogue and
29 Ronald Dworkin, ‘A Matter of Principle’ (Oxford: Clarendon Press,1986), p. 191.
30 Ibid.
31 John Rawls, ‘Political Liberalism’ (New York: Colombia University Press,1993),p.134.
32 ibid.,p. 135.
P a g e | 18
compromise in the spirit of tolerance or irrelevance,33
pluralistic concessions are perhaps
affordable and there are good practical reasons to legitimize such attitudes. It is by these
assumptions that led to political and legal frameworks which lay emphasis on an
accommodative approach in handling religious related disputes in public spheres in hopes of
achieving equality – this approach will henceforth be referred to as “Equality-by-
Accommodation”.
But religion is not homogeneous and because attached to its doctrines is a necessity
for ‘interpretation’ and ‘translation’, its social and worldviews will be ever-changing. As Toni
Massaro aptly summarizes it:
[S]ome religious faiths reject secular reason and pluralism outright as proper cultural
baselines; and fidelity to most religious faiths implies conduct, not just belief or
expression. Official support of religion therefore can mean reinforcement of conduct
that undermines foundational liberal principles of critical inquiry, religious and
ideological pluralism, and secular reason.34
It is when religion is socially or politically perceived as aggressive, that the accommodative
attitude toward it has to change: “[T]he state’s duty [is] to protect its citizens against
excessive demands of individual religious groups.”35
Adverting the risk of granting a public
presence to practices which hold values contrary to a jurisdiction’s liberal constitution led to
the evolution of a more “aggressive” liberal secularism which excludes religious influence
from the public sphere – this will hence forth be referred to as “Neutrality-by-Exclusion”.
33 The Augustinian approach of Catholicismto secular power is not troublingto the worldly power. Reinhold
Niebuhr and many others prefer reason to revelation when itcomes to moral truth. Reinhold Niebuhr, ‘Moral
Man and Immoral Society: A Study of Ethics and Politics’ 28 (2001).These are easy cases for accommodation of
religion.
34 Massaro,supra note17, at 943.
35 Karl-HeinzLadeur & Ino Augsberg, ‘The Myth of the Neutral State: The Relationship Between State and
Religion in the Face of New Challenges’, 8 German Law Journal 143,148 (2007).
P a g e | 19
4. Two Prongs of Liberalism – “Neutrality-by-Exclusion” vs “Equality-by-
Accommodation”
Religious conflicts in the seventeenth century manifested from a theological
disagreement36
. Today, the contention stems from political disagreement. They are conflicts
about whether religious people can bring their religion into the public sphere. In the United
States of America, non-establishment is constitutionally protected and religious pluralism is
at the foundation of the state.37
Whereas in Europe, there still lie many states with an
established church and fairly homogeneous societies38
. However, European societies are
becoming increasingly pluralistic, making conflicts in this region more visible. After
discussing the historical context of liberalism, the dissertation will proceed to dissect further
the two variations of the evolution of liberalism.
The first view which holds that there is only one conception of the good ‘to be
recognised by all citizens who are reasonable and rational’39
will be represented from an
aggressive liberal secular perspective of “neutrality-by-exclusion” where the ‘one conception
of good’ postulated earlier would be the benefit of the society as a whole as exemplified in
the the legitimate aim of ‘living together’ followed by Laicite.
The second, which is essentially a response to the former, will be represented from
the a “equality-by-accommodation” perspective that embraces that ‘there are many
conflicting reasonable comprehensive doctrines with their conceptions of the good, each
compatible with the full rationality of human persons, so far as that can be ascertained with
the resources of a political conception of justice.’40
Raz denies that a government can have
no conception of what the good life is. The good life, is a life of autonomous choices from a
36 J. Ratzinger, ‘Truth and Tolerance: Christian Belief and World Religions’ (Ignatius Press,2004)
37 M. Nussbaum,‘Liberty of Conscience : In Defense of America’s Tradition of Religious Equality’ (Basic Books,
2008)
38 Andorra, Armenia, Denmark, UK Church of England (since Toleration Act 1689, c.13) and Church of Scotland
(Church of Scotland Act 1921), Finland,Georgia,Greece, Iceland,Liechtenstein, Malta,Monaco, Norway
39 John Rawls, Political Liberalism (New York: Colombia University Press,1993),p. 134.
40 ibid.,p. 135.
P a g e | 20
range of valuable options, and it is the role of the government to support and facilitate this
multiplicity of views. This side will be discussed in light of the Canadian legal system which
juxtaposed to the French, takes a more accommodative approach. By analysing this
juxtaposition of approaches in the two legal systems, the dissertation evaluates the
effectiveness of a liberal secular legal system which excludes religious influence from the
public sphere in achieving equality.
4.1. “Neutrality-by-Exclusion”: Religion as a Threat
Representatives of aggressive secularism like Andras Sajo41
are principally worried
about religious movements that directly challenge secular arrangements. Andras Sanjo fears
that religion may undermine ‘the legal arrangements that claim to be neutral and generally
applicable to all people living in the national community’.42
The challenge is that religion
forces secular legal systems to agree on compromises and concessions that imperil the
integrity and coherence of secular laws.43
An example would be the growing encroachment
on the freedom of expression for the sake of protecting religious sensibilities. Andras Sanjo
argues that secularism is weak.
The saga of Danish Jyllands-Posten case44
regarding religiously provocative
cartoons shows that secularism is open to compromises and concessions that secular states
should firmly refuse to make. He opines that religion does not have a place in the public
sphere and thus welcomed the abolition of the blasphemy law in the UK.45
However, as far
as the UK is concerned, the abolition does not make it aggressively secular. The abolition of
their blasphemy law must be read in conjunction with the Racial and Religious Hatred Act
41 Sajo A., ‘Preliminaries to a Concept of Constitutional Secularism’,Int’l Const L, (2008),605
42 Ibid
43 Zucca,L., ‘Secular Europe : Law and Religion in the European Constitutional Landscape’, (1st edn, Oxford
University Press,2012).
44 Henkel, Heiko (Fall 2010). "Fundamentally Danish? The Muhammad Cartoon Crisis as Transitional Drama"
(PDF). Human Architecture: Journal of the Sociology of Self-knowledge. 2 VIII.Retrieved 26 April 2016.
45Sajo A., ‘Preliminaries to a Concept of Constitutional Secularism’,Int’l ConstL, (2008), 611 n.13 (referringto
the Criminal Justiceand Immigration Act,2008,c.4, par 79 (Eng))
P a g e | 21
2006, which still limits free speech with the right to be protected from hatred. In section 29J,
after having defined what amounts to an act which stirs up religious hatred. The act states:
“Nothing in this Part shall be read or given effect in a way which prohibits or restricts
discussion, criticism or expressions of antipathy, dislike, ridicule, insult or abuse of
particular religions or the beliefs or practices of their adherents, or of any other belief
system or the beliefs or practices of its adherents, or proselytising or urging
adherents of a different religion or belief system to cease practising their religion or
belief system. “46
This act is an exemplification of what Lorenzo Zucca coins as presumption of
priority.47
While the right to free speech sets the tone for the context we express ourselves,
the fallacy lies in the assumption that our words have rights to absolute freedom. Acts like
these carve out egregious exceptions which nonetheless limits our expression if it intends to
provoke religious hatred. However, the criteria as to falls into this category of expression
remains arbitrary and it is this arbitrariness that makes the legal system vulnerable to bias
and inconsistency where it concerns the right to the freedom of expression.
In Europe, free speech and secularism seem to play a more major role in society and
comparative to the UK, there exists a presumption in favour of liberty. However, the courts
seem to limit the freedom of expression as well, when confronted with the issue of
blasphemy. In the seminal case of Otto Preminger Institute v Austria48
, the applicant institute
tried to show a film that offended the Catholic religion and the religious feelings of the people
of Tyrol, a region that consists of a large majority of Catholics in whose lives religion plays a
very important role. The ECtHR was faced with the decision of whether the administrative
sanction preventing the screening of the movie was in breach of Art.10 of the ECHR, or if it
46 Racial and Religious Hatred Act 2006,section 29J
47Zucca,L., ‘Secular Europe : Law and Religion in the European Constitutional Landscape’, (1st edn, Oxford
University Press,2012). S2.2.1.
48 (13470/87) [1994] ECHR 26 (20 September 1994)
P a g e | 22
was justified on grounds of protecting religious feelings. The court argued that the sanction
was justified as the film risked provocation of a predominantly Christian population.
Critics elucidate the fact that there should be no tension between free speech and
the right to protection from being offended in a religious context, because strictly speaking,
the latter is not a right.49
The point of contention lies in the fact that should such a right exist,
it must apply to feelings in general, not only religious cases- the corollary of which would be
a legal system that would undermine the essence of the right of free speech and expression.
Where other forms of association claim equal protection, what warrants, as a matter of
principle, special treatment when it comes to religious sensitivity? While it is an arduous task
to answer this on political grounds, it is not difficult to justify it on a prudential one. The crux
lies in the fact that because the nature of religiosity demands the deepest of attachments
and identity, religious people are particularly susceptible to offence and are keen on
responding to any perceived offence with any means, legal or otherwise.
4.1.1. “Aggressive Secularism“- Laicite
The best illustration of aggressive secularism is the French notion of Laicite. It is
characterized by two separate elements: legal Laïcité and ideological Laïcité. The former
consists of ‘a very strict separation of church and state, against the backdrop of political
conflict between the state and the Catholic Church that resulted in a law regulating very
strictly the presence of religion in the public sphere (1905)’.50
The latter ‘claims to provide a
value system common to all citizens by expelling into the private sphere.51
A clear
exemplification of its application is the French legislature’s passing of a law prohibiting the
wearing of most religious affiliated symbols, clothing, and garb in public schools.52
49 See Dworkin, R. ‘The Right to Ridicule’, NY Rev Books (23 March 2006),29
50 Id at xii.
51 Id at xiv.
52 Law No. 2004-228 of March 15, 2004, Journal Officiel dela R~publiqueFran~aise [J.O.][Official Gazette of
France], March 17, 2004,p. 5190
P a g e | 23
4.1.2. The State and Religious Exclusion from the Public Sphere:The Burqa Ban
Controversy
This dissertation will use the seminal case of S.A.S. v. France53
to explore the
attitude and approach of the French with regards to neutrality and religion. In S.A.S. v.
France54
, the ECHR upheld on July 1, 2014, the law to ban the wearing of clothing to
conceal one’s face in public spheres.
The applicant is a French national and resident born in 1990. As a devout Muslim,
she wears the burqa and niqab for her religion, culture and personal convictions not out of
coercion from anyone. She concedes that necessary circumstances like identity checks for
security purposes justifies unveiling herself but otherwise, it is within her rights to express
her religiosity and culture as there existed not any intention to annoy anyone else.
The applicant essentially presented three submissions to the Court. Firstly, that the
blanket ban on full-veil clothing in public spheres interferes with her rights under Articles 9
(right to freedom of thought, conscience and religion) and 10(freedom of expression) of the
ECHR. It also trespasses on her right to respect for private life. Secondly, it was argued that
the ban violated her right to freedom of association under Article 11(freedom of peaceful
assembly and to freedom of association with others) alongside Article 14 (the prohibition of
discrimination). Thirdly, that the ban should be viewed in light of Article 3 (the prohibition of
torture or degrading treatment)55
.
The freedom to privacy and freedom of religion or belief (Article 8 &9) was the
dominant and recurrent theme in this case. The Court decided that, in adopting the law, the
government had remained within the restrictions on Article 9(1) which are permitted by
Article 9(2).
53 App. No. 43835/11,2014 Eur. Ct. H.R.
54 Ibid.
55 This will notbe delved into as itis not relevant to the dissertation hypothesis
P a g e | 24
The Court accepted two of the legitimate aims submitted by the government. They
were the grounds of the protection of public safety and protection of the rights and freedom
of others. The Court expressed concerns about the severe consequences for women who
wished to wear the veil and the extent of Islamophobia arguments hidden within the wider
French debate regarding this law.56
However, the Court conceded that the State had
grounds to find the law essential in upholding the requirements of “living together”57
in the
French society. Concealing the face in public could be considered by the State as adversely
affecting the “principle of interaction between individuals, which in its view is essential for the
expression not only of pluralism, but also of tolerance and broadmindedness without which
there is no democratic society”58
. The Court accepted the link between the aim to “the
respect for the minimum requirements of life in society” and the protection of rights and
freedom of others. Viewed from this prospective, the Court had to afford the State a wide
margin of appreciation. This judgement reflects the legal applicability of the “neutrality-by-
exclusion” model.
Further reaffirming this fact, is that ECtHR is undeniably conservative when it comes
to granting relief on issues involving Article 9 ECHR. For instance in Mann Singh v France59
,
a Sikh man was denied his driving license as he had a turban on and identity regulation
required a bareheaded photograph.
In cases that involved removal of religious clothing in the educational environment
like Dablab v Switzerland60
(teachers) or Leyla Şahin v Turkey61
, Kurtulmus v Turkey62
(students) for example, the Strasbourg Court has declared them either inadmissible or
justified under Article 9(2) of the ECHR. The distinction of these cases lie in that there was a
56 Proportionality should only be considered when there was a general threat to public safety,the court
rejected that there was a presence of this threat.
57 Id. atpp.142
58 Id. atpp 56
59App. No. 24479/07,2008 Eur. Ct. H.R
60App. No. 42393/98, 2001 Eur. Ct. H.R.
61App. No. 44774/98,2005 Eur. Ct. H.R.
62App. No. 65500/01,2006 Eur. Ct. H.R.
P a g e | 25
conflict of religious freedom vis-à-vis duties attached to a social post. In this case, the choice
to manifest personal religious convictions in the form of clothing should only be made during
the applicant’s personal time.
The only exception to this “harsh” judgement pattern is in the case of Ahmet Arslan
and Other v Turkey63
, wherein Turkish nationals were convicted for wearing religious
garments in public areas. The court granted relief in this case citing a breach of Article 9.
The ECtHR was aware of the similarities between Ahmet Arslan and the present case, but
were contented with the factual distinction that the present case involved a concealment of
the face and accepted the “living together” principle.
The S.A.S. v France case concluded the prevalent debate about the legality of the
impugned law on banning face-concealing clothing. The Court has reasserted its margin of
appreciation doctrine, showing that it was ready to interpret the Convention in the light of
restrictive domestic approach to religious freedom. Although courts should not shy away
from countering legislative majorities, it should nevertheless respect the sovereignty of the
State, especially when said legislation has gathered unanimous support.
Although unpopular, the Court’s endorsement of the French Republican approach to
their restrictions on religious freedom elucidates an acceptance of their political and legal
structure. The French jurisprudence follows a mix of Kant and Rousseau, positing that
human beings are endowed with reason but do not always use it. It seeks not the just, but
the good.
Regardless of how the French Laïcité contradicts one’s personal conception of
morality, the fact remains that the epistemology behind French jurisprudence follows that
“Rights are not natural, but politically constructed”. 64
They can therefore be subject to
63App. No. 41135/98, 2010 Eur. Ct. H.R.
64 MilletFrancois-xavier ,'European Constitutional LawReview 2015 Case Comment' [2015] 1(1) When the
European Court of Human Rights encounters the face: a case-note on the burqa ban in France
<http://login.westlaw.co.uk/maf/wluk/app/document?&srguId=ia744d06500000151b361b42e1005b8a1&doc
P a g e | 26
significant restrictions in the name of the common good of the political community. Although
this approach does not attest to classical liberalism, it is an approach which is most
compatible with the very ‘universalistic’ spirit of human rights underlying the Convention. A
plethora of legal restrictions have been plagued by religious influence in ‘classically liberal’
societies. For example, restricting rights of homosexuals is undoubtedly a corollary of
religious-politics and their biased conception of ‘evil’. In France however, same-sex marriage
was recognized and passed regardless of the legislatures own personal religious beliefs.
While concerns of islamophobia, gender discrimination or a general infringement of
rights, are to an extent, relevant considerations; Laïcité, as are other political maxims, should
be respected as a tool designed to manage diversity. Personal convictions, religious or
otherwise, should be distinguished from the legal validity of a State’s political regime.
In pragmatic terms, the choice of constitutionalism is a choice birthed from
negotiation and allows for critical democratic discussions of the implementation of legislative
law. However, as evidenced by Europe’s history with ecclesial law, when a religious
organization comes into power, laws would not be of the people’s choice but made based
solely of the religious leader’s interpretation of their doctrine. Even in the impossible case
where all people voluntarily subscribe to a single religion, it cannot be said that policies from
this religion are sovereign choices of the people. The choices will be that of (to a great extent
pre-existing, taken for granted, non-negotiated) religion.
The issue here is not that aggressive secular states like France do not comprehend
religious arguments and their demands for special treatment. However, it draws a clear line
between where the normative authority should lie. The fact is, the choice of secular
constitutional arrangement is intellectually and practically available to all, but this is not true
about an individual’s private religious beliefs.
guId=I27D676D0799611E582588114149BF5E6&hitguId=I27D676D0799611E582588114149BF5E6&rank=6&sp
os=6&epos=6&td=21&crumb-action=append&context=4&resolvein=true> accessed 20 April 2016
P a g e | 27
4.1.3. Secularism vs. Secularization
Andras Sajo supports a constitutional secularism that is aggressive (close to Laicite)
capable of responding to religion and its presence in the public sphere. However, he
bemoans the fact that we lack a strong normative theory of secularism to underpin our legal
systems in order to eradicate such arbitrariness within legal systems. We learn that perhaps
the problem of achieving equality lies with the perception of secularism itself.
At this point, it is important to understand the difference in ‘secularization’ and
‘secularism’. Andras Sajo regards secularism as a social fact which stands for an ambiguous
social reality, and that it is uncertain as a legal concept. Secularization on the other hand, is
defined by Sajo as ‘a historical project still in the making’.65
For Sajo, it is about religion and
its organizations conceding its power over various aspects of life in favour of the State.66
However, regarding secularization as a historical project is deemed problematic.
Olivier Roy defines secularization as ‘a social phenomenon that requires no political
implementation’.67
This holds the implication that secularization is not a power struggle
between state and church, but a process of the gradual waning of religion in society. In this
case, secularism and secularization may go hand in hand theoretically, with secularization
as a process and the notion of secularism as a political project with a set of normative claims
concerning the way in which the state deals with diversity. Practically, however, the
resurgence of religion, raises doubts as to the direction of the political project and the
facticity of the secularization social process.
Zucca propounds that a comparative constitutional theory of secularism faces the
problem that the notion of secularism is deeply intertwined with the local practices and
histories in the west, as Charles Taylor has powerfully demonstrated.68
Andras Sajo is
65 Sajo A., ‘Prelimanaries to a Concept of Constitutional Secularism’,Int’l ConstL, (2008),609
66 Ibid.
67 Roy, O. Secularism Confronts Islam (Columbia University Press,2007),7
68 Taylor,C. A Secular Age (Harvard University Press,2007)
P a g e | 28
similarly aware of the importance of social history and other contingencies in the formation of
the Western understanding of secularism, explaining its weakness as that it is a ‘fuzzy
constitutional concept’.
According to Sajo, the project of secularization has never been coherently conceived
and brought forward in the majority of legal and political systems. If secularism is weak and
uncertain because of its local rootedness, secularization seems to be half hearted
compromises by the church that are coincidently thought to be compatible with
secularization itself. Andras Sajo argues that the project of secularisation is weak because it
fails to display the intellectual consistency required to free the public sphere from religion.
The crux of this argument lies in the perception of secularization, if one subscribes to Olivier
Roy’s claim at secularization is not an intellectual project but an organic development of a
society in response to the gradual waning of religion in people’s life, secularization would
then mirror gradual development and thus, by definition, it seems to be an ideal that is never
meant to be achieved.
4.1.4. Analysis: Secularism (Aggressive) is a Stronger Ideal than Perceived
The author of this dissertation would like to point out certain doubts with regards to
this perception of secularism. There is an inherent contradiction in ascertaining that the
source of secularism’s uncertainty is its local rootedness and claiming that the process of
secularization is an organic development of the waning of religion.
There seems to be a perceived consensus that secularization is inconsistent
because it is dependent on the cultural history of a jurisdiction. While it is undeniably true
that domestic culture and history determines a society’s moral and political zeitgeist, what
seems to have been overlooked is the universality of the shift from faith to reason. While our
ancestors lived with faith “naively,” we can live with or without it “reflectively.” By contrast, we
P a g e | 29
now live in a society in which, for the first time in history, “a purely self-sufficient humanism
has come to be a widely available option.”69
Critics purport, that religious conflict is a symptom of mismanagement of pluralism
within a secular state, not a threat to equality. However, this dissertation propounds that the
accommodation of religion itself is the biggest obstacle to the ideal of equality.
The cause for strong religious attachment lies in the fact that religion was at its roots
manifested in regions that lack pluralism and diversity. It is precisely due to this fact that
religious extremism seems to stem from jurisdictions, communities or even social groups
which lack diversity. This explains the wane of religious influence over populations as time
passes; globalisation has gradually imposed onto societies pluralism and diversity. The
corollary of which is an environment which allows for intellect, reason and critical thinking to
be nurtured, qualities of which are the building blocks for equality and peaceful co-existence.
Over-sensitivity, extremism and blind adherence to religious doctrine – the causes of
religious conflicts; stem from an absence of these qualities.
The author hence propounds that secularisation should be viewed as an organic
development or as Olivier Roy puts it, a ‘social phenomenon’ toward the ideal that is
secularism- the intellectual ideal of achieving true equality void of obstacles caused by
religion. By viewing it in such manner, we will overlook not the universal nature of the
secularisation process and understand that while it may be organic, there still lies a need for
positive actions (by governance) in order to achieve this ideal.
Another flaw in the discussion earlier lies in Sajo’s definition of secularism as a social
fact which stands for an ambiguous social reality - that it is uncertain as a legal concept. This
dissertation concedes that it aptly depicts the state of secularism in most of today’s world.
However, the ambiguity and uncertainties are only present when governance, influenced by
political ambition or religious bias, do not take a strong enough stand in excluding all aspects
69 Ibid,p.18
P a g e | 30
of society which have a propensity to incite discrimination and undermine equality – racism,
sexism, religiosity etc.
The theoretical validity and competence of secularism as a political and legal
construct should not be dismissed with reasons of the likes of poor practical application. It is
due to this that the dissertation propounds that “aggressive secularism” which practices
“neutrality-by-exclusion”, if properly enforced, is the best approach toward achieving
equality.
P a g e | 31
4.2. “Equality-by-Accommodation”
“Open Secularism” - Canada
In order to fully comprehend secularism, it is essential to analyse the two variants
ascribed to it, the first as discussed earlier, “aggressive secularism”, and the other “open
secularism”. These competing visions of secularism were at the forefront of the highly
publicized Bouchard-Taylor Commission70
that was constituted in Quebec, Canada to
investigate the issue of accommodation practices in light of it being a pluralistic, democratic
and egalitarian society. According to the commission, the four key principles constituting any
model of secularism are: the moral equality of persons; freedom of conscience and religion;
State neutrality towards religion; and the separation of Church and State.
“Aggressive secularism” accords more importance to neutrality rather than to
freedom of conscience and religion, attempting to relegate the religiosity to the private
sphere, leaving the public sphere as “a neutral ground that stands outside religious
controversy”.71
Contrastingly, “open secularism” is based on the protection of religion, even if this
requires a relaxation of the principle of neutrality. In this model, state neutrality towards
religion and the separation of Church and State are not seen as ends in themselves, but
rather as the means to achieving the fundamental objectives of respect for religious and
moral equality and freedom of conscience and religion.72
In “open secularism”, priority is
given in favour of religious freedom and equality when there is any contradiction or tension
between the various constituent facets of secularism. The crux lie in that this conception of
70 Named as such for the Commission‟s co-chairs,Gérard Bouchard and Charles Taylor.Its formal titleis the
Consultation Commission on Accommodation Practices Related to Cultural Differences
71 Moon, R. “Introduction: Law and Religious Pluralism in Canada,” Law and Religious Pluralismin Canada,ed.
Richard Moon (Vancouver: UBC Press,2008): 6
72 Jose Woehrlingand RosalieJukier, 'Religion and the Secular State in Canada' [2010] 1(1) Religion and The
Secular State 185
P a g e | 32
secularism is directed at state institutions rather than individuals and does not strive to
neutralise religion as an identity marker in society.
The “open secularism” stance is represented aptly by Chief Justice Dickson in the
seminal case of R v Big M Drug Mart73
:
“[a] truly free society is one which can accommodate a wide variety of beliefs, diversity of
tastes and pursuits, customs and codes of conduct.”74
The Canadian courts apply this in their interpretation of the Canadian Charter of Rights and
Freedoms75
. A more detailed review of Canada’s “open secularism” and a comparison to the
French “aggressive secularism” will provide a more nuanced picture of the relationship
between religion and the secular state. In doing so we will be able to competently evaluate
the dissertation subject question of whether “[a] liberal secular legal system which excludes
religious influence in the public sphere would adequately respond to the challenge of
achieving equality”.
73 [1985] 1 S.C.R. 295
74 Id, at para.94,18 D.L.R. (4th) 321
75 Canadian Charter of Rights and Freedoms, Part I of the Constitution Act, 1982, being Schedule B to the
Canada Act 1982 (U.K.), 1982,c.11 [hereinafter Charter]. See José Woehrling,“The Open Secularism Model of
the Bouchard-Taylor Commission Report and the Decisions of the Supreme Court of Canada on Freedom of
Religion and Religious Accommodation,” Religion,Culture and the State – Reflections on the Bouchard-Taylor
Commission,eds. Howard Adelman and PierreAnctil (Toronto: University of Toronto Press,forthcoming in
2010).
P a g e | 33
4.2.2. The State and Religious Autonomy: Analysis of the Kirpan Controversy
In Canada, the state and religious communities operate in separate spheres and, in
principle, it is not the role of the state, or secular courts, to intervene in their organization, nor
to interfere in their autonomous governance.76
As Justice Iacobucci of the Supreme Court of
Canada stated, “the State is in no position to be, nor should it become, the arbiter of
religious dogma.”77
A clear juxtaposition in approaches to the attitude of the separation of Church and
State can be seen in the reactions toward the French headscarf controversy78
. The French
President Jacques Chirac, in response, characterized the law as necessary to maintain the
religiously neutral nature of French schools and the tenet of secularism (laicite), or strict
separation of church and state.79
However, the Supreme Court of Canada (S.C.C)
responded in saying that it “"sent a strong message that Canada's public education
institutions must embrace diversity and develop an educational culture respectful of the right
to freedom of religion."80
In order to fully understand this ‘equality-by-accommodation’ approach taken by
Canada, the case of Multani v. Commission scolaire Marguerite-Bourgeoys81
and the
Canadian courts treatment of the kirpan wearing issue will be delved into.
In Multani, the S.C.C. held that an orthodox Sikh student could wear a kirpan, a
ceremonial dagger, to school given his strong religious beliefs. The Canadian courts have
76Woehrling,J. and Jukier, R. 'Religion and the Secular State in Canada' [2010] 1(1) Religion and The Secular
State 190
77 Amselem, supra n. 41 at para 50.
78 Also known as “l'affaire dufoulard”
79 Press Release, U.S. Commission on Int'l Religious Freedom, “France: Proposed Bill May Violate Freedom of
Religion” (Feb. 3, 2004), availableat
http://www.uscirf.gov/mediaroom/press/2004/february/02032004_france.html; Walterick,supra note3, at
252; R. Murray Thomas, “Religion in Schools: Controversies in The World” 30-31 (2006) (statingthat purpose of
laiciteis to ensure citizen loyalty to the state and the principles of freedom and neutrality).B
80 Terrance S. Carter and Anne-Marie Langan, “Supreme Court Gives Strong Endorsement to Freedom of
Religion”, Church Law Bulletin No. 17 (Carters Prof I Corp., Orangeville,Ontario,Can.),Mar. 16, 2006,at 1,
availableathttp://www.carters.ca/pub/bulletin/church/200 6/chchlb 17.pdf
81 [2006] 1 S.C.R. 256,
P a g e | 34
addressed the issue of kirpan wearing in a number of lower court cases82
. In Tuli v. St. Albert
Protestant Separate School District No. 683
, the plaintiff sought an injunction against the
school board's policy that would have suspended or expelled him for wearing his kirpan to
school.84
In a short decision, the court authorized the kirpan if blunted and held tightly within
its sheath while on school property.85
The court found that allowing the kirpan would "provide
those who are unfamiliar with the tenet of his faith an opportunity to be introduced to and to
develop an understanding of another's culture and heritage."86
In Ontario Human Rights Comm. v. Peel Bd. of Education87
, found that a total ban of
wearing kirpans in school was problematic as there is a lack of evidence suggesting that it
posed a danger.88
The courts rejected the Peel Board of Education’s no-weapons policy with
regards to categorizing the kirpan as a dangerous weapon rather than a religious symbol.
The Board was unable to reach an acceptable compromise with affected members of the
Sikh community such as wearing a replica or stitching the kirpan into its sheath to prevent its
removal.89
In its judgement, the court considered that there were no reported incidents of
school violence associated with kirpans but neglected the fact that at the time there had
been incidents of violence associated with kirpans outside of the school setting including two
stabbings.90
If the propensity of danger posed by the kirpan determines its categorization as
a ‘dangerous weapon’, should there have been a distinction between acts of violence
occurring in school or outside the premises?
82 Hothi v. R., [1985] 3 W.W.R. 256 (Can.), represents a prominent caseoutsideof the school context. In that
case,the Manitoba Court of Queen's Bench upheld a provincial judge's decision to bar a Sikh defendant from
wearing a kirpan in the courtroom duringhis trial.Seeid. 11. "
83 [1985] 8 C.H.R.R. D/3906 (Can.)
84 Id. ; BARNETT, supra note 5, at12.
85 Id. 7-8; BARNETT, supra note 5, at 12
86 Id. 4
87 [1991] 80 D.L.R. (4th) 475,
88 Id. at476.
89 Id. at476-77.
90 Id. at 477 See also Hamilton,supra note15, at 115 (citinga number of kirpan-related violentacts in Canada)
P a g e | 35
Regardless, in light of the accommodative disposition of the Canadian courts, the
court rejected a prohibition on the wearing of kirpans in schools by students and teachers
upon consideration of the Ontario Human Rights Code 1981.91
The issue of kirpan in schools
did not arise again in the courts until Multani reached the S.C.C. almost fifteen years later.92
In Multani, the S.C.C. unanimously agreed to set aside a court of appeal decision
upholding a school board Council of Commissioners' prohibition on a student's kirpan
wear.93
The issue began in late 2001 when Gurbaj Singh Multani's kirpan fell from his
clothing when playing on a playground at Ecole Sainte-Catherine-Laboure, a French
language school in Montreal. 94
Baptized Sikhs, like the Multanis, believe in five symbols of
faith (the Five K's) including a comb (kangha), a pair of britches (kachha), a bracelet (karha),
a head turban to cover uncut hair (keski), and a sword (kirpan).95
Sikhs view the kirpan as a
religious symbol to be worn at all times rather than a weapon.96
But the school governing
board eventually found that wearing a kirpan on school property would violate the school
code of conduct's prohibition on weapons.97
Quebec's Attorney General intervened, issuing
a statement emphasizing a zero-tolerance policy for "knives" in school, including kirpans.98
The S.C.C in deciding this case identified two issues. First, the prohibition from
wearing his kirpan at school constituted an infringement of the student’s rights under the
Quebec Charter or the Canadian Charter of Rights and Freedoms; Second, whether this
91 Id. at 480
92 WilliamJ.Smith, “Private Beliefs and Public Safety: The Supreme Court Strikes Down a Total Ban on the
Kirpan in Schools as Unreasonable”, 16 EDUC. & L.J. 83, 83 (2006) (Can.)
93 Multani v. Commission scolaire Marguerite-Bourgeoys, [2006] 1 S.C.R. 256 (Can.); Canada Allows Sikh
Students to Carry Daggers, Church and State, Apr. 1, 2006,at 21, availableat
http://www.av.org/site/News2?page=NewsArticle&id8087&abbr--cs.
94 Stuart Laidlaw, “Accommodating Religions Challenges Secular Society; Supreme Court Shows Leadership with
Recent Ruling Urges Schools to Teach About Their Students' Religions”, Toronto Star (Can.), Mar. 11,2006,at
L09; WilliamJ. Smith, “BalancingSecurity and Human Rights: Quebec Schools Between Pastand Future”, 14
Edu. & L.J. 99, 111 (2004) (Can.)
95 Mann, supra note 33, at 61-62;Mcloed, supra note 33, at 32
96 Timeline: The Quebec Kirpan Case, supra note 31.
97 Multani,[2006] 1 S.C.R. 256, 4; Smith, supra note 31, at 111-12
98 Smith, supra note 28, at 93. Smith's analysis suggests theAttorney General's comments might have
stemmed from broader public safety concerns such as 9/11 and terrorism.Smith, supra note 31,at 126-27
P a g e | 36
infringement was justified under an exception in the Quebec Charter.99
The S.C.C held that
there was an infringement on the student’s freedom of religion under S.2(a) of the Charter.100
The court acknowledged the presence of internal limits in S.2(a) which states that public
safety could justify restricting a person’s freedom to practice religious beliefs, however, they
declined to find such limitations applicable in this case.101
In R. v. Oakes102
, the S.C.C. set out the criteria used to evaluate restrictions on rights
under the Charter. To qualify as a constitutional infringement of a Charter right, the
government imposition must stem from an important objective, be rationally connected to an
important objective, minimize impairment on an individual's rights, and embody
proportionality between the objective sought and the imposition on the individual.103
In
Multani, this test was used to consider the validity of the kirpan ban, however, the S.C.C
focused more on accommodation and found a lack of evidence to support a total ban on
kirpan wearing.104
In reaching its conclusion, the court noted "a total prohibition against wearing a
kirpan to school undermines the value of this religious symbol and sends students the
message that some religious practices do not merit the same protection as others."105
The case review of Multani elucidates Canada’s accommodative attitude toward
freedom of religion, thus, the 1982 Canadian Charter of Rights and Freedoms remains a
99 Multani,[2006] 1 S.C.R. 256, at 13; Smith, supra note 31, at 115. An additional issueinvolved determining
the proper standard of review for reviewing the school board's decision in an administrativecontext. See
Smith, supra note 28, at 93-94.The majority of the court agreed to apply the stringent standard of review used
in constitutional lawcases,buttwo minority opinions argued for an administrativelawstandard
100 Id at 33; 33; Bussey, supra note 31, at2. A more in-depth treatment of S. 2(a) follows.
101 Smith, supra note 28, at 96-97; Multani,[2006] 1 S.C.R. 256, 26
102 [1986] 1 S.C.R. 103 (Can.)
103 Id.; Luan-Vu N. Tran, “The Canadian Charter of Rights and Freedoms: Justification, Methods, and Limits of a
Multicultural Interpretation”, 28 Column. HUM. RTs. L. REV. 33,58-59 (1996).
104 Smith, supra note 28, at 104. The minority opinion expresses concern with applyingaccommodation
doctrines to public concerns.See id at 102.Because the court found a total ban unreasonable,itdid not have
to consider the proportionality prongof the Oakes test. Id.at 105.
105 Multani,[2006] 1 S.C.R. 256, at 79.
P a g e | 37
core document protecting individual liberties and expression.106
It serves to ensure priority is
given to human dignity over the imposition of a standardized national
ideology.107
Accordingly, under S.1, freedoms and rights articulated in the Charter are
guaranteed "subject only to such reasonable limits prescribed by law as can be
demonstrably justified in a free and democratic society."108
When considering religious freedom in the Canadian jurisdiction, the following
provisions must be scrutinized. Firstly, s.2 of the Charter characterizes freedom of
conscience and religion as a fundamental freedom held by "[e]veryone."109
This refers only
to the expressly granted freedoms of religion, expression, press, and association, whereas
the United States conception of fundamental freedoms is based on a larger collection of
rights having either an express or implied basis in the U.S. Constitution.110
On the other
hand, in France, freedom of religion is not fundamental, but rather, rejection of religious
belief is seen as fundamental. Scholars theorize that France's recent acceptance of the
weight of religious obligations may explain the country's difficulty in accepting religious
difference.111
Second, s.15 requires that every individual be treated equally by and under the law
"without discrimination based on race, national or ethnic origin, colour, [or]
106 Horwitz, supra note 51, at 3 (acknowledgingthat the preamble to the Canadian Charter serves as a
reminder that "a citizen may have two sources of obligation");see also Tran,supra note 47, at 35 (callingthe
Charter a "promisingdocument capableof mediating opposingvalues and interests").
107 Tran, supra note 47, at 50, 54
108 Canadian Charter of Rights and Freedoms, Part I of the Constitutional Act,1982, being Schedule B to the
Canada Act 1982, ch. 11 (U.K.), s.1.
109 Canadian Charter of Rights and Freedoms s. 2, PartI of the Constitution Act, 1982,being Schedule B to the
Canada Act 1982, ch. 11 (U.K.)
110 Tran, supra note 47, at 42-43.
111 See Alain Garay,BlandineChelini-Pont,Emmanuel Tawil & Zarah Anseur, “The Permissible Scope of Legal
Limitations on the Freedom of Religion or Belief in France”, 19 Emory International LawReview. 785, 819
(2005).
P a g e | 38
religion . . . ."112
Finally, s. 27 calls for an interpretation of the Charter in a manner that
preserves and enhances the "multicultural heritage of Canadians”.113
4.2.3. Analysis: Conceptual Flaws of the “Accommodative” Approach to Equality
Canadian courts' treatment of religious freedom cases often begins with and is best
exemplified by R. v. Big M Drug Mart114
; it acknowledges that "in certain contexts minority
communities suffer considerable disadvantages, even threats to their existence, if they are
treated as identical to the majority group.”115
While the facticity of this statement cannot be
argued, it simply accentuates the flaws in this accommodative approach towards achieving
equality.
There are essentially two conceptual flaws. First is the State’s concession to there
never being a solution to the segregation of the population. While population segregation is
organic especially due to the rise of pluralism in societies, it remains in essence the
antithesis of equality amongst the population. One of the most crucial factors that cause
social segregation, a casual factor of religious conflict, is the empowerment of contradictory
religious dogma- which realistically can never be proportionate. If social segregation is at the
root of the equality puzzle, can a legal approach that concedes to and even encourages this
political and sociological flaw even be included in the equality conversation?
Critics of secularism have a tendency to accentuate the main problem with the
equality puzzle: neutrality is impossible. Secularists have no claim to neutrality because
everyone has a set of presuppositions that guide their moral and ethical analyses.
112 Canadian Charter of Rights and Freedoms S. 15(1),Part I of the Constitution Act, 1982,being Schedule B to
the Canada Act 1982, ch. 11 (U.K.).
113 Id. s.27
114 [1985] 1 S.C.R. 295 (Can.)
115 Tran, supra note 47, at 62.
P a g e | 39
Contending for any position carries with it these presuppositions. It is unrealistic to claim that
anyone interprets facts with zero bias where the mind is a ‘black slate’.
The crux of this critique lies in the claim that there can be no neutral ground when
taking a stance regarding the “human condition”116
and hence secularism no matter how
aggressive cannot claim to be neutral. Any opinions and positions would inevitably be
“normative, value-laden, metaphysical in character”117
For example, in the mid-1990s, the
Human Embryo Research Panel established by the National Institutes of Health in the
United States of America, where they discussed their deliberations on the federal funding of
research on human embryos, declared, the “Panel weighed arguments for and against
Federal funding of this research in light of the best available information and scientific
knowledge and conducted its deliberations in terms that were independent of a particular
religious or philosophical perspective.”118
However, Meilaender clarified the position of the opposing argument when he
asserted, “We are not philosopher-kings who can adjudicate disputes between conflicting
views without ourselves being parties to the argument.”119
The stance is premised on the
assumption that we simply are unable to enter into bioethical discourse and leave our pre-
suppositional framework at the door and hence it is argued that secularism cannot claim to
be neutral.
The author of this dissertation concedes that within every claim is premised upon
some pre-suppositional framework. It is however, a fallacy to claim that because of the
presence of a pre-suppositional framework, that secularism is not neutral.
116 The human condition is defined as "the characteristics,key events, and situations which composethe
essentials of human existence, such as birth,growth, emotionality,aspiration,conflict,and mortality."
117 Meilaender,G. “Bioethics and the Character of Human Life,” The New Atlantis:1 (Spring
2003)http://www.thenewatlantis.com/archive/1/meilaender.htm (accessed April 25,2016).
118 Ibid.
119 Ibid.
P a g e | 40
The distinguishing factor lies in the nature of these presuppositions. The argument
for the exclusion of religion from the public sphere revolves around the fact that contradictory
sets of stone-age religious dogma are asserting factual scientific claims with non-scientific
evidence. These claims which directly affect the legal society, for example - the prohibition of
stem cell research due to the Christian claim that it would constitute the killing of ‘souls’, are
essentially premised upon faith based interpretative claims from a specific variant amongst
the plethora of different non-evidential religious based claims of ‘absolute truth’.
The standard of proof to any religious claim is hence, the blind faith in what is
preached in an individual’s choice of a specific religious doctrine out of the variety of
contradictory dogmas. In this case, it is undoubtedly true that pre-suppositional frameworks
of this nature can never by neutral.
On the contrary, in a liberal secular state which excludes religious influence, any
factual claim would be held to a high standard of empirical evidence, intellectual debate and
critical thinking – values of which can be said to be universal and hence foundationally
neutral. If neutrality is purported to be an unachievable goal because any opinion would
inevitably be “normative, value-laden, metaphysical in character”, then a liberal secular
society which excludes religion is undoubtedly the best, most feasible method in achieving
neutrality. A legal system void of religious influence in the public sphere would by nature, be
one that strives to defeat these obstacles to neutrality. The insistence of an empirical,
evidential and reasonable standard of proof to any claim, stems from scientific objectivity
free from the arbitrariness of metaphysical claims.
In fact, the secular position is one that is closest to being devoid of these pre-
suppositions. It is by nature not a position that lays claim to any absolute truth, but a
mechanism run by universalistic intellectual criteria to facilitate the democratic reaching of
consensuses free from unsubstantiated claims to absolute truths.
P a g e | 41
Secondly, there seems to be an assumption that the only alternative to
accommodating the religious minority group is an adherence to the belief of the majority.
This fallacy, which presupposes a necessity of bias, is again one that fundamentally
contradicts the notion of neutrality and equality. An insistence on ‘choosing a side’, by
definition implies that a group has been favoured over another. It is due to this that the
author of this dissertation supports a more aggressive ‘neutrality-by-exclusion’ secularism. It
is also integral to note that there should be no claim that choosing secularism over religiosity
is in itself a bias. The contradictory nature of religious dogma relative to each other makes
achieving equality by means of accommodation not only conceptually flawed but realistically
unattainable. The “equality-by-accommodation” position is one that embraces the assertion
of “religious absolute truths”- the embracing of which, relative to other religions, is a
manifestation of bias.
On the contrary, a secular position which excludes any form of religious influence
from the public, is one that strives to achieve equality without any assertions of any “absolute
truths”. It is one with a raised standard to proof (empirical, evidential and scientific) when it
comes to the evaluation of any factual claims. These two positions are not comparable
because the former embraces the assertions of “absolute truths” which are inherently
biased, while the former makes no assertions but instead, as previously discussed,
implements higher standards of proof as a system of check and balance – a neutral position.
P a g e | 42
5. Conclusion
Throughout history the place and role of religion in the public sphere has been
generally addressed through a binary model, scrutinizing the juxtaposition between secular
and religious. In order to achieve equality and be truly neutral towards different religious (and
non-religious) conceptions of life, the fact is, the public sphere cannot but be secular. Public
decisions affecting economics, politics and law must be reason-based not faith-based in
order to sustain public order as ordered liberty. Constitutionalism exists only where political
powers do not ground their public affecting decisions on transcendental concerns. Andras
Sajo aptly describes this by the example, “People are buried in cemeteries not because it
facilitates resurrection but for public health reasons.”120
Aggressive secularism as an institutional arrangement and legal framework is a
shield for reason-based polity121
against a social “(dis)order” based on dictates of religious
doctrine. This makes it the best response to the challenge of achieving equality. It is crucial
to understand that the purpose of the aggressive secular position propounded is not to just
exclude religion; exclusion is but the means to achieving equality. Secularism is about a form
of communication accessible to all (believers or non-believers), relying on empirical,
substantiated critical discussion. It provides a universal standard for judging what deserves
normative authority. It is illogical for one to claim that the sensitivity that follows the nature of
religious attachment deserves any special treatment. Religious sensitivity is, in a way,
related to identity. It does not command specific legal respect.122
The opposite would be an
abuse of the exceptionalism of free exercise.
The insistence of a reason-based political and legal society dictates that legal
choices be based on secular public reasons – which are reasons accessible to all,
120 Andras,S. ‘Constitutionalism and Secularism: The Need For Reason ' [2009] 30(6) Cardozo Law Review 2401
121 an organized political community livingunder a singlesystem of government of which policies are
determined by reason
122 András Sajó, “The crisis that was not there: Notes on a reply”, IntJ Constitutional Law(2009) 7 (3): 515-528
firstpublished onlineJune15, 2009 doi:10.1093/icon/mop014
P a g e | 43
regardless of their religious belief. Religiously grounded reasons require an element of
“interpretation”. Without secularism’s insistence on reason-giving, faith-based divine
command accessible only to the believer will potentially become acceptable and even
mandatory.
Allowing religious influence in law and the public sphere, undermines on a
fundamental level, the principles of constitutionalism and most relevantly and particularly –
equality. If popular sovereignty presupposes that all members of the community have
reasoning capacity that enables them to participate in political decision-making, ‘blind’ faith
by definition undermines it conceptually. Critics of secularism stand by ‘the inability of the
secular state to cope with the fact of pluralism’. However, as Sajo states, ‘it is not the secular
or non-secular nature of the state that disables its capacity to deal with pluralism’. If ideals,
religious or otherwise, threaten the cohesion of our polities, then it becomes the concern of
the shared epistemological foundations and the possibility of communication within the
constitutional political and legal community. It is only with epistemological unity that enables
peaceful and equal co-existence.
The fact is, the aggressive secular position does not propose exclusion of religion
simply in order to cope with disagreement. The issue is that religious groups, especially the
major organized ones, are not concerned with disagreement. They seek exclusive imposition
of their divinely commanded positions. That is why the author consistently accentuates the
fact that an aggressive secular system is the best way to achieve equality. There will no
doubt be objections by the religious groups and secularism offers space for disagreement,
but as the dissertation has clearly established, religious-based demands should be strictly be
confined to the private sphere.
Equality, unlike what many critics purport, is not an unachievable ideal. But it is one
that requires the competence of governing body to properly enforce the “neutrality-by-
exclusion” concept and the endurance to manage any collateral conflict. It is one that
P a g e | 44
requires time. Time for the population to gradually understand that the relegation of religion
into the private sphere is for the betterment of the legal, political and social dynamics of the
society. The dissertation has established that accommodating to religious demands is
conceptually incompatible with any notion of equality, it has showed the legal validity and
dismissed the misconceptions regarding the uncertainty and ambiguity of the aggressive
secular position. But most importantly, this dissertation has established that a secular
system that excludes religious influence from the public sphere is the best way to achieve
equality.
[Word Count:10,486]
P a g e | 45
Bibliography
Table of Cases
 Ahmet Arslan and Other v. Turkey, Application No. 41135/98, 2010 Eur. Ct. H.
 Braunfeld v. Brown, 366 U.S. 599 (1961)
 Dahlab v. Switzerland, Application No. 42393/98, 2001 Eur. Ct. H.R.
 Hothi v. R., [1985] 3 W.W.R. 256 (Can.)
 Kurtulmus v Turkey, Application No. 65500/01, 2006 Eur. Ct. H.R.
 Mann Singh v. France (dec.), App. No. 24479/07, 2008 Eur. Ct. H.R
 Multani v. Commission scolaire Marguerite-Bourgeoys [2006] 1 S.C.R. 256,
 Otto Preminger Institute v Austria (13470/87) [1994] ECHR 26 (20 September 1994)
 Ontario Human Rights Comm. v. Peel Bd. of Education [1991] 80 D.L.R. (4th) 475
 Reynolds v. United States, 98 U.S. (8 Otto.) 145 (1878)
 R. v. Oakes [1986] 1 S.C.R. 103 (Can.)
 R. v. Big M Drug Mart [1985] 1 S.C.R. 295 (Can.)
 R v Big M Drug Mart [1985] 1 S.C.R. 295
 S.A.S. v. France [GC], App. No. 43835/11, 2014 Eur. Ct. H.R.
 Sahin v. Turkey, Application No. 44774/98, 2005 Eur. Ct. H.R.
 Tuli v. St. Albert Protestant Separate School District No. 6 [1985] 8 C.H.R.R. D/3906
(Can.)
Table of Legislation
 Church of Scotland Act 1921
 Canadian Charter of Rights and Freedoms, Part I of the Constitution Act, 1982, being
Schedule B to the Canada Act 1982 (U.K.), 1982
 Criminal Justice and Immigration Act 2008
 European Convention of Human Rights Art.3,8,9,10,14
P a g e | 46
 Law No. 2004-228 of March 15, 2004, Journal Officiel de la R~publique Fran~aise
[J.O.][Official Gazette of France], March 17, 2004,
 Racial and Religious Hatred Act 2006
 Toleration Act 1689
Secondary Sources
Books
 Adams J., ‘A Defence of the Constitutions of Government of the United States of
America’ (3rd edn, The Lawbook Exchange Ltd, 1787)
 Dworkin R., ‘A Matter of Principle’ (Oxford: Clarendon Press, 1986),
 Dworkin R., ‘The Right to Ridicule’, NY Rev Books (23 March 2006)
 Kymlicka W., ‘Multicultural Citizenship: A Liberal Theory of Minority Rights’, (Oxford:
Oxford University Press 1995), 1995.
 Locke J., ‘A Letter Concerning Toleration and Other Writings'. (Indianapolis: Liberty
Fund, 2012).
 Ludwig Von Mises, ‘Liberalism’ (1st edn, The Foundation for Economic Education,
Inc Irvington-on-Hudson, New York 10533 and Cobden Press 2002)
 Roy O., Secularism Confronts Islam (Columbia University Press, 2007)
 Raz J., ‘The Morality of Law’ (Oxford: Clarendon Press, 1989)
 Rawls J., ‘Political Liberalism’ (New York: Colombia University Press, 1993)
 Sajo A., ‘Preliminaries to a Concept of Constitutional Secularism’, Int’l Const L,
(2008),
 Zucca L., ‘A Secular Europe: Law and Religion in the European Constitutional
Landscape’ (1st
edition, Oxford University Press 2012)
P a g e | 47
Journals
 Basil Mitchell, ‘Law, Morality, and Religion in a Secular Society’ (Oxford: Oxford
University Press, 1967
 Bader V., “Secularism or Democracy? Associational Governance of Religious
Diversity”(2007)
 Bentham J., ‘A Fragment on Government.’ (London, 1776),
 Dieter Grimm, 'General Laws and Religious Norms' [2009] 30(6) Conflicts Between
General Laws and Religious Norms
 Ferrari S., 'Law and religion in a secular world: a European
perspective' [2012] 1(1) Ecclesiastical Law Journal
 Galanter M., ‘Religious Freedoms in the United States: A Turning Point’ [1966]
Wisconsin Law Review 2
 Garay A., Chelini-Pont B., Tawil E. & Anseur Z., “The Permissible Scope of Legal
Limitations on the Freedom of Religion or Belief in France”, 19 Emory International
Law Review. 785, 819 (2005).
 Hosen N. and Mohr R., Law and Religion in Public Life (London, 2011).
 Habermas J., ‘Notes on a Post-secular Society’ (2008)
 Henkel H., (Fall 2010). "Fundamentally Danish? The Muhammad Cartoon Crisis as
Transitional Drama" (PDF). Human Architecture: Journal of the Sociology of Self-
knowledge. 2 VIII. Retrieved 26 April 2016.
 Karl-Heinz Ladeur & Ino Augsberg, ‘The Myth of the Neutral State: The Relationship
Between State and Religion in the Face of New Challenges’, (8 German Law Journal
2007)
 Laidlaw S., “Accommodating Religions Challenges Secular Society; Supreme Court
Shows Leadership with Recent Ruling Urges Schools to Teach About Their Students'
Religions”, Toronto Star (Can.), Mar. 11,2006
P a g e | 48
 Luan-Vu N. Tran, “The Canadian Charter of Rights and Freedoms: Justification,
Methods, and Limits of a Multicultural Interpretation”, 28 Column. HUM. RTs. L. REV.
33, 58-59 (1996).
 Meilaender G., “Bioethics and the Character of Human Life,” The New Atlantis: 1
(Spring 2003)http://www.thenewatlantis.com/archive/1/meilaender.htm (accessed
April 25, 2016).
 Mill, John Stuart. ‘On Liberty’. (London: Longman, Roberts & Green, 1869);
 Millet Francois-xavier , 'European Constitutional Law Review 2015 Case Comment'
[2015] 1(1) When the European Court of Human Rights encounters the face: a case-
note on the burqa ban in France
<http://login.westlaw.co.uk/maf/wluk/app/document?&srguId=ia744d06500000151b3
61b42e1005b8a1&docguId=I27D676D0799611E582588114149BF5E6&hitguId=I27
D676D0799611E582588114149BF5E6&rank=6&spos=6&epos=6&td=21&crumb-
action=append&context=4&resolvein=true> accessed 20 April 2016
 Moon R., “Introduction: Law and Religious Pluralism in Canada,” Law and Religious
Pluralism in Canada, ed. Richard Moon (Vancouver: UBC Press, 2008):
 Murray Thomas R., “Religion in Schools: Controversies in The World” 30-31 (2006)
 Nussbaum M., ‘Liberty of Conscience : In Defense of America’s Tradition of Religious
Equality’ (Basic Books, 2008)
 Niklas Luhmann, ‘Observations on Modernity’ (1st
edn, Stanford University
Press 1998)
 Press Release, U.S. Commission on Int'l Religious Freedom, “France: Proposed Bill
May Violate Freedom of Religion” (Feb. 3, 2004), available at
http://www.uscirf.gov/mediaroom/press/2004/february/02032004_france.html;
 Peter Griffith, 'Christianity, liberalism and the separation of law and
morals' [2001] 1(1) UCL Jurisprudence Review
P a g e | 49
 Raphael Cohen-Almagor, “Ethnocultural Minorities in Liberal Democracies”, in: R.
Cohen-Almagor (ed.), Challenges to Democracy: Essays in Honour and Memory of
Isaiah Berlin, London: Ashgate Publishing Ltd., 2000
 Reinhold Niebuhr, ‘Moral Man and Immoral Society: A Study of Ethics and Politics’
28 (2001)
 Ratzinger J., Truth and Tolerance: Christian Belief and World Religions (Ignatius
Press, 2004)
 Sajó A, ‘Constitutionalism and Secularism: The Need For Reason
' [2009] 30(6) Cardozo Law Review 2401
 Sajó A., “The crisis that was not there: Notes on a reply”, Int J Constitutional Law
(2009) 7 (3): 515-528 first published online June 15, 2009 doi:10.1093/icon/mop014
 Terrance S. Carter and Anne-Marie Langan, “Supreme Court Gives Strong
Endorsement to Freedom of Religion”, Church Law Bulletin No. 17 (Carters Prof I
Corp., Orangeville, Ontario, Can.), Mar. 16, 2006, at 1, available at
http://www.carters.ca/pub/bulletin/church/200 6/chchlb 17.pdf
 Tourkochoriti I., ‘The Burqa Ban: Divergent Approaches to Freedom of Religion in
France and in the USA’ 20(3) William and Mary Bill of Rights Journal (2012)
 Taylor C., A Secular Age (Harvard University Press,2007)
 Woehrling J., “The Open Secularism Model of the Bouchard-Taylor Commission
Report and the Decisions of the Supreme Court of Canada on Freedom of Religion
and Religious Accommodation,” Religion, Culture and the State – Reflections on the
Bouchard-Taylor Commission, eds. Howard Adelman and Pierre Anctil (Toronto:
University of Toronto Press, forthcoming in 2010).
 Woehrling J. and Jukier R., 'Religion and the Secular State in
Canada' [2010] 1(1) Religion and the Secular State
P a g e | 50
 William J. Smith, “Private Beliefs and Public Safety: The Supreme Court Strikes
Down a Total Ban on the Kirpan in Schools as Unreasonable”, 16 EDUC. & L.J. 83,
83 (2006) (Can.)
 William J. Smith, “Balancing Security and Human Rights: Quebec Schools Between
Past and Future”, 14 Edu. & L.J. 99, 111 (2004) (Can.)

Weitere ähnliche Inhalte

Andere mochten auch

Essay 4, excellent effort realizing full credit of 10 of 10 possible pts.
Essay 4, excellent effort realizing full credit of 10 of 10 possible pts.Essay 4, excellent effort realizing full credit of 10 of 10 possible pts.
Essay 4, excellent effort realizing full credit of 10 of 10 possible pts.jcarlson1
 
Lewis presentation-- SECULARIZATION AND ITS IMPACT ON HABITUS FORMATION
Lewis presentation-- SECULARIZATION AND ITS IMPACT ON HABITUS FORMATIONLewis presentation-- SECULARIZATION AND ITS IMPACT ON HABITUS FORMATION
Lewis presentation-- SECULARIZATION AND ITS IMPACT ON HABITUS FORMATIONjim lewis
 
Publicly funded Islamic Education- Berglund
Publicly funded Islamic Education- BerglundPublicly funded Islamic Education- Berglund
Publicly funded Islamic Education- BerglundJenny Berglund
 
ECHR grand chamber judgment s.a.s v. france ban on wearing in public clothin...
ECHR grand chamber judgment s.a.s v. france  ban on wearing in public clothin...ECHR grand chamber judgment s.a.s v. france  ban on wearing in public clothin...
ECHR grand chamber judgment s.a.s v. france ban on wearing in public clothin...REITER LEGAL
 
What makes Muslims feel French? (Social Forces)
What makes Muslims feel French? (Social Forces)What makes Muslims feel French? (Social Forces)
What makes Muslims feel French? (Social Forces)UNCEurope
 
A FORUM: DEMOCRACY AND RELIGION Back to the question
A FORUM: DEMOCRACY AND RELIGION Back to the questionA FORUM: DEMOCRACY AND RELIGION Back to the question
A FORUM: DEMOCRACY AND RELIGION Back to the questioneurope_in_israel
 
Freedom of expression in France .
 Freedom of expression in France . Freedom of expression in France .
Freedom of expression in France .Sadek KHEDDACHE
 
Birgit Sauer - Conflicts over values: the issue of Moslem headscarves
 Birgit Sauer - Conflicts over values: the issue of Moslem headscarves Birgit Sauer - Conflicts over values: the issue of Moslem headscarves
Birgit Sauer - Conflicts over values: the issue of Moslem headscarvesARGE Bildungsmanagement
 
Dialogue with Islam: Facing the Challenge of Muslim Integration in France, Ne...
Dialogue with Islam: Facing the Challenge of Muslim Integration in France, Ne...Dialogue with Islam: Facing the Challenge of Muslim Integration in France, Ne...
Dialogue with Islam: Facing the Challenge of Muslim Integration in France, Ne...thinkingeurope2011
 
Changes napoleon did
Changes napoleon didChanges napoleon did
Changes napoleon didmardalex
 
French vs us school sys
French vs us school sysFrench vs us school sys
French vs us school sysjs430167mhs
 
Assignment 2: class debate
Assignment 2: class debateAssignment 2: class debate
Assignment 2: class debate마 이환
 
France politics
France politicsFrance politics
France politicsboldeagle3
 
CONSTITUTIONAL SECULARISM AND FREEDOM OF RELIGION IN NIGERIA; ANALYSIS OF ASI...
CONSTITUTIONAL SECULARISM AND FREEDOM OF RELIGION IN NIGERIA; ANALYSIS OF ASI...CONSTITUTIONAL SECULARISM AND FREEDOM OF RELIGION IN NIGERIA; ANALYSIS OF ASI...
CONSTITUTIONAL SECULARISM AND FREEDOM OF RELIGION IN NIGERIA; ANALYSIS OF ASI...Russel Eraga
 

Andere mochten auch (20)

Essay 4, excellent effort realizing full credit of 10 of 10 possible pts.
Essay 4, excellent effort realizing full credit of 10 of 10 possible pts.Essay 4, excellent effort realizing full credit of 10 of 10 possible pts.
Essay 4, excellent effort realizing full credit of 10 of 10 possible pts.
 
Lewis presentation-- SECULARIZATION AND ITS IMPACT ON HABITUS FORMATION
Lewis presentation-- SECULARIZATION AND ITS IMPACT ON HABITUS FORMATIONLewis presentation-- SECULARIZATION AND ITS IMPACT ON HABITUS FORMATION
Lewis presentation-- SECULARIZATION AND ITS IMPACT ON HABITUS FORMATION
 
Publicly funded Islamic Education- Berglund
Publicly funded Islamic Education- BerglundPublicly funded Islamic Education- Berglund
Publicly funded Islamic Education- Berglund
 
ECHR grand chamber judgment s.a.s v. france ban on wearing in public clothin...
ECHR grand chamber judgment s.a.s v. france  ban on wearing in public clothin...ECHR grand chamber judgment s.a.s v. france  ban on wearing in public clothin...
ECHR grand chamber judgment s.a.s v. france ban on wearing in public clothin...
 
What makes Muslims feel French? (Social Forces)
What makes Muslims feel French? (Social Forces)What makes Muslims feel French? (Social Forces)
What makes Muslims feel French? (Social Forces)
 
A FORUM: DEMOCRACY AND RELIGION Back to the question
A FORUM: DEMOCRACY AND RELIGION Back to the questionA FORUM: DEMOCRACY AND RELIGION Back to the question
A FORUM: DEMOCRACY AND RELIGION Back to the question
 
Unit 8 Canada Today
Unit 8   Canada TodayUnit 8   Canada Today
Unit 8 Canada Today
 
Freedom of expression in France .
 Freedom of expression in France . Freedom of expression in France .
Freedom of expression in France .
 
Birgit Sauer - Conflicts over values: the issue of Moslem headscarves
 Birgit Sauer - Conflicts over values: the issue of Moslem headscarves Birgit Sauer - Conflicts over values: the issue of Moslem headscarves
Birgit Sauer - Conflicts over values: the issue of Moslem headscarves
 
Religious diversity in France
Religious diversity in FranceReligious diversity in France
Religious diversity in France
 
Dialogue with Islam: Facing the Challenge of Muslim Integration in France, Ne...
Dialogue with Islam: Facing the Challenge of Muslim Integration in France, Ne...Dialogue with Islam: Facing the Challenge of Muslim Integration in France, Ne...
Dialogue with Islam: Facing the Challenge of Muslim Integration in France, Ne...
 
Changes napoleon did
Changes napoleon didChanges napoleon did
Changes napoleon did
 
Dissertation
DissertationDissertation
Dissertation
 
France
FranceFrance
France
 
French vs us school sys
French vs us school sysFrench vs us school sys
French vs us school sys
 
Assignment 2: class debate
Assignment 2: class debateAssignment 2: class debate
Assignment 2: class debate
 
Diss-updated
Diss-updatedDiss-updated
Diss-updated
 
France politics
France politicsFrance politics
France politics
 
CONSTITUTIONAL SECULARISM AND FREEDOM OF RELIGION IN NIGERIA; ANALYSIS OF ASI...
CONSTITUTIONAL SECULARISM AND FREEDOM OF RELIGION IN NIGERIA; ANALYSIS OF ASI...CONSTITUTIONAL SECULARISM AND FREEDOM OF RELIGION IN NIGERIA; ANALYSIS OF ASI...
CONSTITUTIONAL SECULARISM AND FREEDOM OF RELIGION IN NIGERIA; ANALYSIS OF ASI...
 
Kingdom Secular
Kingdom SecularKingdom Secular
Kingdom Secular
 

Ähnlich wie Dissertation Final 15012951

Communication and Exchange in Secular and Catholic Discourse
Communication and Exchange in Secular and Catholic DiscourseCommunication and Exchange in Secular and Catholic Discourse
Communication and Exchange in Secular and Catholic DiscourseRuairidh MacLennan
 
Review the articles, Women and Financialization Microcredit, Inst.docx
Review the articles, Women and Financialization Microcredit, Inst.docxReview the articles, Women and Financialization Microcredit, Inst.docx
Review the articles, Women and Financialization Microcredit, Inst.docxronak56
 
A SHORT HISTORY OF ETHICS
A SHORT HISTORY OF ETHICSA SHORT HISTORY OF ETHICS
A SHORT HISTORY OF ETHICSLisa Muthukumar
 
RESEARCH PROPOSAL3Research proposalCompare and contra.docx
RESEARCH PROPOSAL3Research proposalCompare and contra.docxRESEARCH PROPOSAL3Research proposalCompare and contra.docx
RESEARCH PROPOSAL3Research proposalCompare and contra.docxgholly1
 
Polarization: On the Threshold between Political Ideology and Social Reality
Polarization: On the Threshold between Political Ideology and Social RealityPolarization: On the Threshold between Political Ideology and Social Reality
Polarization: On the Threshold between Political Ideology and Social RealityUniversité de Montréal
 
Social change (Social System)
Social change (Social System)Social change (Social System)
Social change (Social System)EHSAN KHAN
 
An Exposition Of The Morality Of Abortion (A Catholic Church Position
An Exposition Of The Morality Of Abortion (A Catholic Church PositionAn Exposition Of The Morality Of Abortion (A Catholic Church Position
An Exposition Of The Morality Of Abortion (A Catholic Church PositionSarah Adams
 
Essay 1 generally good content; but some issues with content as n.docx
Essay 1 generally good content; but some issues with content as n.docxEssay 1 generally good content; but some issues with content as n.docx
Essay 1 generally good content; but some issues with content as n.docxYASHU40
 
Nathalie English Research.docxFZXCXCXZCZXXZC
Nathalie English Research.docxFZXCXCXZCZXXZCNathalie English Research.docxFZXCXCXZCZXXZC
Nathalie English Research.docxFZXCXCXZCZXXZCLyndon32
 
Chernobyl Essay. Nuclear Power Accident: Chernobyl Free Essay Sample on Samp...
Chernobyl Essay.  Nuclear Power Accident: Chernobyl Free Essay Sample on Samp...Chernobyl Essay.  Nuclear Power Accident: Chernobyl Free Essay Sample on Samp...
Chernobyl Essay. Nuclear Power Accident: Chernobyl Free Essay Sample on Samp...Britney Gilbert
 
Playing with Fire_ An Ethnographic Look at How Polyamory Functio
Playing with Fire_  An Ethnographic Look at How Polyamory FunctioPlaying with Fire_  An Ethnographic Look at How Polyamory Functio
Playing with Fire_ An Ethnographic Look at How Polyamory FunctioMaleia Mikesell
 
Natural World Order & The Islamic Thought
Natural World Order & The Islamic ThoughtNatural World Order & The Islamic Thought
Natural World Order & The Islamic ThoughtShafiq Khan
 
Critique Essay Examples. J 176: Media Fluency for the Digital Age: Example of...
Critique Essay Examples. J 176: Media Fluency for the Digital Age: Example of...Critique Essay Examples. J 176: Media Fluency for the Digital Age: Example of...
Critique Essay Examples. J 176: Media Fluency for the Digital Age: Example of...Kelly Simon
 
A global ethic taha abderrahman
A global ethic   taha abderrahmanA global ethic   taha abderrahman
A global ethic taha abderrahmanMohamed Akklouch
 
A global ethic taha abderrahman
A global ethic   taha abderrahmanA global ethic   taha abderrahman
A global ethic taha abderrahmanMohamed Akklouch
 
PHIL 212 Introductory Ethics Module 3Meta-ethical Questions.docx
PHIL 212 Introductory Ethics  Module 3Meta-ethical Questions.docxPHIL 212 Introductory Ethics  Module 3Meta-ethical Questions.docx
PHIL 212 Introductory Ethics Module 3Meta-ethical Questions.docxmattjtoni51554
 
Outline of the TheoristsKarl Marx Turned Hegel on his head .docx
Outline of the TheoristsKarl Marx Turned Hegel on his head .docxOutline of the TheoristsKarl Marx Turned Hegel on his head .docx
Outline of the TheoristsKarl Marx Turned Hegel on his head .docxalfred4lewis58146
 
A Third Culture Kid Theology (2016)
A Third Culture Kid Theology (2016)A Third Culture Kid Theology (2016)
A Third Culture Kid Theology (2016)Bryce Nelson
 

Ähnlich wie Dissertation Final 15012951 (20)

Communication and Exchange in Secular and Catholic Discourse
Communication and Exchange in Secular and Catholic DiscourseCommunication and Exchange in Secular and Catholic Discourse
Communication and Exchange in Secular and Catholic Discourse
 
Workbook i tae2
Workbook i tae2Workbook i tae2
Workbook i tae2
 
Review the articles, Women and Financialization Microcredit, Inst.docx
Review the articles, Women and Financialization Microcredit, Inst.docxReview the articles, Women and Financialization Microcredit, Inst.docx
Review the articles, Women and Financialization Microcredit, Inst.docx
 
A SHORT HISTORY OF ETHICS
A SHORT HISTORY OF ETHICSA SHORT HISTORY OF ETHICS
A SHORT HISTORY OF ETHICS
 
RESEARCH PROPOSAL3Research proposalCompare and contra.docx
RESEARCH PROPOSAL3Research proposalCompare and contra.docxRESEARCH PROPOSAL3Research proposalCompare and contra.docx
RESEARCH PROPOSAL3Research proposalCompare and contra.docx
 
Polarization: On the Threshold between Political Ideology and Social Reality
Polarization: On the Threshold between Political Ideology and Social RealityPolarization: On the Threshold between Political Ideology and Social Reality
Polarization: On the Threshold between Political Ideology and Social Reality
 
Social change (Social System)
Social change (Social System)Social change (Social System)
Social change (Social System)
 
An Exposition Of The Morality Of Abortion (A Catholic Church Position
An Exposition Of The Morality Of Abortion (A Catholic Church PositionAn Exposition Of The Morality Of Abortion (A Catholic Church Position
An Exposition Of The Morality Of Abortion (A Catholic Church Position
 
Sociology Topics For Essay
Sociology Topics For EssaySociology Topics For Essay
Sociology Topics For Essay
 
Essay 1 generally good content; but some issues with content as n.docx
Essay 1 generally good content; but some issues with content as n.docxEssay 1 generally good content; but some issues with content as n.docx
Essay 1 generally good content; but some issues with content as n.docx
 
Nathalie English Research.docxFZXCXCXZCZXXZC
Nathalie English Research.docxFZXCXCXZCZXXZCNathalie English Research.docxFZXCXCXZCZXXZC
Nathalie English Research.docxFZXCXCXZCZXXZC
 
Chernobyl Essay. Nuclear Power Accident: Chernobyl Free Essay Sample on Samp...
Chernobyl Essay.  Nuclear Power Accident: Chernobyl Free Essay Sample on Samp...Chernobyl Essay.  Nuclear Power Accident: Chernobyl Free Essay Sample on Samp...
Chernobyl Essay. Nuclear Power Accident: Chernobyl Free Essay Sample on Samp...
 
Playing with Fire_ An Ethnographic Look at How Polyamory Functio
Playing with Fire_  An Ethnographic Look at How Polyamory FunctioPlaying with Fire_  An Ethnographic Look at How Polyamory Functio
Playing with Fire_ An Ethnographic Look at How Polyamory Functio
 
Natural World Order & The Islamic Thought
Natural World Order & The Islamic ThoughtNatural World Order & The Islamic Thought
Natural World Order & The Islamic Thought
 
Critique Essay Examples. J 176: Media Fluency for the Digital Age: Example of...
Critique Essay Examples. J 176: Media Fluency for the Digital Age: Example of...Critique Essay Examples. J 176: Media Fluency for the Digital Age: Example of...
Critique Essay Examples. J 176: Media Fluency for the Digital Age: Example of...
 
A global ethic taha abderrahman
A global ethic   taha abderrahmanA global ethic   taha abderrahman
A global ethic taha abderrahman
 
A global ethic taha abderrahman
A global ethic   taha abderrahmanA global ethic   taha abderrahman
A global ethic taha abderrahman
 
PHIL 212 Introductory Ethics Module 3Meta-ethical Questions.docx
PHIL 212 Introductory Ethics  Module 3Meta-ethical Questions.docxPHIL 212 Introductory Ethics  Module 3Meta-ethical Questions.docx
PHIL 212 Introductory Ethics Module 3Meta-ethical Questions.docx
 
Outline of the TheoristsKarl Marx Turned Hegel on his head .docx
Outline of the TheoristsKarl Marx Turned Hegel on his head .docxOutline of the TheoristsKarl Marx Turned Hegel on his head .docx
Outline of the TheoristsKarl Marx Turned Hegel on his head .docx
 
A Third Culture Kid Theology (2016)
A Third Culture Kid Theology (2016)A Third Culture Kid Theology (2016)
A Third Culture Kid Theology (2016)
 

Dissertation Final 15012951

  • 1. P a g e | 1 “A liberal secular legal system which excludes religious influence in the public sphere would adequately respond to the challenge of achieving equality…” In view of secular liberalism’s understanding of equality and its critique, how accurate is this statement? Brandon Toh Kah Meng April 2016 Dissertation submitted in partial fulfilment for the degree of LLB. Law (Hons) Law University of Hertfordshire
  • 2. P a g e | 2 Abstract The rise of religious pluralism has triggered much debate and an ambiguous academic response vis-à-vis the best approach toward achieving equality. This dissertation addresses an ambitious research subject area: Does religion have a place in a secular public sphere and what are the effects of its exclusion and inclusion on achieving equality in a religiously diverse society. This problem is tackled by scrutinizing the two different ends of the spectrum of secularism – “Aggressive”and “Open” Secularism.Aggressive secularism will be discussed in light of Laïcité and the Burqa Ban Case1 under the category of “Neutrality-by-Exclusion”while Open secularism will be studied in light of the Canadian legal system and the seminal case of Multani2 under the category of “Equality-by-Accommodation”. By analysing both positions, pointing out conceptual flaws and debunking misconceptions, the dissertation establishes that an exclusion of religious influence from the public sphere is the best way to achieve equality. 1 S.A.S. v. France [GC], App. No. 43835/11,2014 Eur. Ct. H.R. 2 Multani v. Commission scolaire Marguerite-Bourgeoys [2006] 1 S.C.R. 256,
  • 3. P a g e | 3 Acknowledgements I would like to extend my sincere and heartfelt gratitude towards all the people who have helped and supported me in this ambitious research project. Without their active guidance, help, cooperation and encouragement. I would not have made headway in the project. I am especially thankful to Dr. Camille Pommel for her conscientious guidance and encouragement. The complexity of my research subject area required extended periods of consultation which she graciously accommodated to. I would also like to extend thanks to Dr. Felipe Romero Moreno,the dissertation course manager, for promptly replying enquiries and for the facilitation of the program. The University of Hertfordshire has provided me with not only this opportunity to challenge my literary capabilities but also a holistic legal education which contributed to my academic capacity – for that I am forever indebted. Lastly, I would also like to acknowledge with a deep sense of reverence, my gratitude towards my mother who has always been there for me and I hope this dissertation will do her proud. SincerelyThanking You Brandon Toh
  • 4. P a g e | 4 Table of Contents Abstract .....................................................................................................................2 Acknowledgements ...................................................................................................3 Table of Contents ......................................................................................................4 1 Introduction...........................................................................................................5 2 Is There A Conflict?: Religion vs. Law and Equality ......................................6-10 3 History of Liberalism...........................................................................................10 3.1 Political Foundations: Birth of Classical Liberalism ...............................10-11 3.2 Liberalism and Democracy..................12-1Error! Bookmark not defined. 3.2.1 Principles of the Liberal Solution... 1Error! Bookmark not defined. 3.3 Liberalism and Utilitarianism.................... 1Error! Bookmark not defined. 3.4 Liberalism and Equality ..........................................................................14-15 3.4.1 Classification of Liberalism.........................................................15-19 4 Two Prongs of Liberalism:"Neutrality-by-Exclusion" vs "Equality-by-Accommodation" 19-20 4.1 "Neutrality-by-Exclusion"........................................................................20-22 4.1.1 “Aggressive Secularism“- Laicite.....................................................22 4.1.2 The Burqa Ban Controversy.....2Error! Bookmark not defined.-27 4.1.3 Secularism vs. Secularization.................................................27-28 4.1.4 Analysis: Secularism is a Stronger Ideal than Perceived ..........28-30 4.2 "Equality-by-Accommodation"................... Error! Bookmark not defined.1 4.2.1 “Open Secularism” - Canada....Error! Bookmark not defined.1-33 4.2.2 The Kirpan Controversy............Error! Bookmark not defined.3-38 4.2.3 Analysis: Conceptual Flaws of the “Accommodative” ApproachError! Bookmark not defined.8-41 5 Conclusion.....................................................................................................42-44 Biblography.........................................................................................................45-50
  • 5. P a g e | 5 1. Introduction The rise of religious pluralism has triggered much debate and an ambiguous academic response vis-à-vis the best approach toward achieving equality. This dissertation addresses an ambitious research subject area: Does religion have a place in a secular public sphere and what are the effects of its exclusion and inclusion on achieving equality in a religiously diverse society. On one side, there is the ‘aggressive’ secular position – often associated with the French Laïcité. This position advocates the strict separation of public and private spheres, relegating religion to the latter. On the other hand, there is the ‘open’ secular position – often associated with the Canadian legal and political system. This position prioritises the accommodation of religious belief and freedoms even if it is at the expense of the principle of neutrality. These juxtaposing positions are the manifestation of legal and political struggles to grapple with a jurisdiction’s constitutional identity and foundational values: ‘rule of law, democracy, human rights, and, last but not least secularism.’3 In our age of increasing religious pluralism, the best way to manage diversity has once again come into question. Equality, the right of different groups of people to have a similar social position and receive the same treatment, has become a complex puzzle that this dissertation will attempt to solve. This will be done by analysing the two secular positions represented above – “Aggressive” and “Open” secularism. The dissertation will first elucidate the problematic relationship between religion and equality. Secondly, study of the development liberalism will first be done to give historical context and an understanding of the underlying principles of these two variations of secularism. Aggressive secularism will be discussed in light of Laïcité and the Burqa Ban Case4 under the category of “Neutrality-by-Exclusion” 3 L. Zucca,‘A Secular Europe: Law and Religion in the European Constitutional Landscape’ (1st edition, Oxford University Press 2012) at1 4 S.A.S. v. France [GC], App. No. 43835/11,2014 Eur. Ct. H.R.
  • 6. P a g e | 6 while Open secularism will be studied in light of the Canadian legal system and the seminal case of Multani5 under the category of “Equality-by-Accommodation”. A critical analysis of the critique of these two positions will entail. Following which, in the conclusion, the dissertation will re-assert the stance that aggressive secularism which excludes religion from the public sphere will most appropriately respond to the challenge of achieving equality. 2. Is there a Conflict?: Religion vs. Law and Equality Religion is a complex, historically and socio-culturally embedded concept. In this section, the dissertation will explore the fundamental reasons why a conflict between law and religion exists. The two main characteristics that make it a formidable obstacle in achieving equality are: Firstly, there is a plethoric variety of religious ideology. Secondly, there is increasing acknowledgment that observations, or claims, of religions are inevitably rooted in competing religious and cultural traditions themselves.6 This seems to exclude any possibility of transcultural or universalistic definitions or second-order observations7 of religions. The spectrum of types of religion reach far beyond the main Abrahamic-beliefs and these contemporaneous types of religions differ in many dimensions: (1) Is the belief centred upon a distinguishability between an immanent and a transcendent - world?8 And if so, which semantics9 ? (2) Are they belief-centred or centred on practices? 5 Multani v. Commission scolaire Marguerite-Bourgeoys [2006] 1 S.C.R. 256, 6 Habermas, J. ‘Notes on a Post-secular Society’ (2008), at 4. 7 “The first-order observer lives in a world that seems both probableand true. By contrast,the second-order observer notices the improbability of first-order observation.”Luhmann, N. ‘Observations on Modernity’ (1st edn, Stanford University Press 1998) 8 Transcendence refers to the aspect of a god's nature and power which is wholly independent of the material universe, beyond all physical laws.This iscontrasted with Immanence, where a god is said to be fully present in the physical world and thus accessibleto creatures in various ways. 9 The study of meaning. It focuses on the relationship between signifiers—likewords,phrases,signs,and symbols—and whatthey stand for, their denotation.
  • 7. P a g e | 7 (3) Is the religious belief transmitted orally (e.g. as mythos) or by written text. (4) Is there a propensity for experts (e.g. rabbis, theologians or sheiks etc) to make their holy scripture consistent and systemic (dogmatising canons, rules and regulations) (5) Is the religious community bound solely by shared practices or beliefs, or does it revolve around a more formalised and hierarchically structured organisation (orders, temples, mosques, churches, congregations) and authoritative rulers (bishops, popes, patriachs)? The corollary of which is a complication in any goal in achieving equality in an increasingly pluralistic world. The recent conditions of increasing religious pluralism seems to be accompanied by a fragmentation of organised religions which gradually insist more vigorously on respect for their belief and on living according to the commandments of their individual creeds in public spheres. The result of which is pressure exerted onto existing forms of institutionalisation of religions and their management by the state. These developments have prompted a reconsideration of the traditional conception of religion in social sciences as well as normative disciplines, especially law and jurisprudence. The essay will focus on the latter and discuss the practical challenges it brings. Religiosity has in every legal jurisdiction to different degrees made its mark. The US legal jurisdiction has been met with famous religious cases concerning polygamy, the flag salute, conscientious objections and other exemptions as well as the use of drugs. The Supreme Court’s mounting difficulties in finding defensible definitions of religion under conditions of greater and manifestly visible religious diversity are excellently analysed by Galanter and in a HLR Note.10 10 Galanter,M. ‘Religious Freedoms in the United States: A Turning Point’, (1966) Wisconsin LawReview 2, at 235 and 260; Note ‘Developments in the Law: Religion and the State’, (1987)100 Harvard LawReview 7, at 1622-1631,1647.On similar difficulties in other countries and international covenants,see V. Bader, Secularism or Democracy? Associational Governance of Religious Diversity (2007),at 301 (note 3).
  • 8. P a g e | 8 In most western jurisdictions, where bias was originally in favour of the national majority religion of Christianity (protestant), the prospective of the majority religion, implicitly or explicitly endorsed by the Court, has been disguised as an objective standard. This is perceptible in Reynolds v. United States11 where a federal law banning polygamy was upheld, undermining the Mormon religion. In Braunfeld v. Brown12 , Pennsylvania courts which required stores to be closed on Sundays, undoubtedly for the benefit of Christian religious practices, held that it was not an unconstitutional interference with religion as described in the first amendment of the United States Constitution even though the Jewish appellants submitted that it unduly burdened them since their religion required them to close their stores on Saturdays as well. European societies too face a growing number of conflicts due to increasingly concerning claims based on freedom of religion. The reason however, is neither a revival of archaic inner-Christian controversies nor a new hostility of the state vis-à-vis religion or a new identification with a single denomination as was the tradition in many European countries. Rather, the conflicts have their source in the growing multiculturalism of European societies, caused by the immigration of members of non-Christian beliefs.13 These beliefs often, alike Christian denominations, lay claim to a divine absolute truth, but unlike Christian denominations, have not undergone the process of historicization and contextualization of these divine revelations that permits the Christian churches to relatively greater extents adopt a more distant approach on the imposition of their values and commandments. Moreover, some religious communities adhere to religious- influenced laws which are blatantly incompatible with our modern conception of human rights. For example, within Sharia law, there are specific forms of punishment for certain crimes. Strict governance 11 98 U.S. (8 Otto.) 145 (1878) 12 366 U.S. 599 (1961 13 D. Grimm, 'General Laws and Religious Norms' [2009] 30(6) Conflicts Between General Laws and Religious Norms 2370
  • 9. P a g e | 9 under Sharia law dictates for example, that fornication is punishable by stoning, the consumption of alcohol by lashing, and theft by the amputation of limbs. The argument for religious autonomy and against religious coercion can be categorized in two groups: inter-group relationships (one group imposing its views on another), and intra-group relationships (a group imposing its views on its own members). The crux lie in that no one group should have the right to coerce the entire society into abiding by its conception of what the law should be based on their religious ideology. For example, some immigrants hide behind the guise of ‘multiculturalism’ as a pretext for imposing patriarchal practices on women or children. This would include stifling the rights of women or even traditional customs such as coercive arranged marriages of sorts.14 This dissertation proposes that the best method in achieving equality is to establish a liberal secular legal system which excludes religious influence in the public sphere because the accommodation of religion fundamentally contradicts the concept of equality. In order to grasp this notion, let us consider a hypothetical scenario where religious minorities are left alone to run their own communities in accordance with their religious commandments. This would entail accepting any contravention of human rights. For instance, parliament would have to ‘respect’ a minority’s illiberal consensus with regard to a woman’s role in society, limiting them from education or holding public office. Kymlicka and Cohen-Almagor argue that if women who dislike this restriction can easily leave the community and enter the larger society, and if the minority group has some historical claim to local self-government, then this may mean that it would be wrong for us to coercively interfere and prohibit that practice. 15 In spite of that, the fact is, wrongful intervention justifies not the moral legitimacy of the practice. 14 For a discussion of conflicts between some EastAsian immigrantgroups and the British government over girls’education and arranged marriages,see Poulter (1987), 589-615;Barber (2004), and Ghimire (2006). 15 Kymlicka,Will, ‘Multicultural Citizenship: A Liberal Theory of Minority Rights’, (Oxford: Oxford University Press 1995).and Raphael Cohen-Almagor, “Ethnocultural Minorities in Liberal Democracies”, in: R. Cohen-
  • 10. P a g e | 10 Can a liberal secular society in the name of equality then exclude the religious belief of the majority from the public sphere, hence undermining the cultural history of the jurisdiction, and the minority, whose members intend to integrate into the community? This dissertation will explore that by first scrutinising the historical context and the synergy of liberal principles that lead to our modern conception of liberal secularism. 3. History of Liberalism 3.1. Political Foundations: Birth of Classical Liberalism Liberal ideas seem to have only surfaced in European politics in the early 16th century, however, liberalism can be traced back to the Middle Ages. In the Middle Ages, the rights and responsibilities of an individual were decided on a hierarchical social system. This feudal stratification of society in the Middle Ages gradually dissolved due to the impact of the slow commercialization and urbanization of Europe at that time, the intellectual ferment of the Renaissance, and the spread of Protestantism in the 16th century. This resulted in a convoluted mess of fear, instability and chaos within the region. The response to this civil dissension at that time, was monarchical absolutism. By the end of the 16th century, the authority of the papacy had been broken in most of northern Europe, and the rulers of different regions resorted to consolidating unity of his realm by enforcing conformity either to Roman Catholicism or to the ruler’s preferred version of the Christian faith (such as Protestantism). The imposing of these juxtaposing schools of belief, albeit of the same religion, onto the governance of these societies culminated in the Thirty Years’ War (1618 – 48). The immense damage to the European region, alongside the fact that no creed managed to eradicate its enemies, led to a general consensus that toleration was the lesser of two evils and that neglecting different beliefs of citizens was inimical to prosperity and good order. Almagor (ed.), Challenges to Democracy: Essays in Honour and Memory of Isaiah Berlin, London: Ashgate PublishingLtd., 2000, 89-118.
  • 11. P a g e | 11 Post 16th century, different industries and commerce expanded and economic policies based on mercantilism, which was an advocacy of government intervention in a country’s economic affairs to increase wealth, were adopted. However, this intervention increasingly served established interest and inhibited enterprise. The newly emerging middle class challenged this. This challenge was a significant factor in the great revolutions that rocked England and France in the 17th and 18th centuries—most notably the English Civil Wars (1642–51), the Glorious Revolution (1688), the American Revolution (1775–83), and the French Revolution (1789). Classical liberalism as an articulated creed is a result of those great collisions. Toleration emerged in the seventeenth century and was depicted as the best response to these religious conflicts. It was recognised a key political virtue which the state imposed as a legal obligation. An example of its legal implementation is the Act of Toleration 168916 . These political ideals were given formal expression n philosophers like John Locke. He regarded toleration as an imposition of reason and the lack thereof is explained in terms of being carried away by ‘irregular positions.’ Both the Act of Toleration and Locke’s ‘Letter Concerning Toleration’17 are examples of a moralizing attitude of the political and intellectual elite of that time. The ability of regard differing beliefs as conditionally acceptable and imposing reason from an ideal moral viewpoint formed the foundations of most early liberal theories. These normative theories rely on the assumption about the fallacy of some religious beliefs and the rightness of some liberal values. These theories form the bedrock of the conception of modern secularism. 16The subtitlesays:‘An Act for exempting their Majesty’s Protestant Subjects dissentingfrom the Church of England from the Penalties of certain Lawes.’ 17 Locke, J., ‘A Letter Concerning Toleration and Other Writings'. (Indianapolis:Liberty Fund, 2012).
  • 12. P a g e | 12 3.2. Liberalism and Democracy In the eighteenth to the early party of the nineteenth century, a political program was formulated from these ideals to serve as a guide to social policy. Early liberals strived to free individuals from religious conformity and aristocratic privilege- social constraints that were preserved by the powers of government. Their aim was to limit the power of government over individual rights but hold them accountable to the governed individual. However, nowhere was this program ever completely carried out. Classical liberalism fell victim to ambivalence, torn between the emancipating tendencies that entailed the revolutions and fears that a universal “majority rules” franchise would undermine private property. John Adams in his Defense of the Constitutions of Government of the United States of America18 accentuated the dangers of allowing the majority to control all branches of government, he declared: “debts would be abolished first; taxes laid heavy on the rich, and not at all on others; and at last a downright equal division of everything be demanded and voted.” Most eighteenth and nineteenth century liberal politicians feared popular sovereignty and for a long period limited suffrage to property owners. In Britain, even the Reform Bill of 186719 held onto property qualifications for the right to vote. In France, only 200,000 out of approximately thirty million citizens were qualified voters despite the ideal of universal male suffrage proclaimed in 1789 and reaffirmed in the Revolutions of 1830. In the United States, it was only until 1860 that universal male suffrage prevailed albeit only for white citizens. Racial and sexual prejudice also served to limit the liberal franchise and in the United States, slavery deprived large numbers of people of freedom. 18 John Adams, A Defence of the Constitutions of Government of the United States of America (3rd edn, The Lawbook Exchange Ltd, 1787) 19 30 & 31 Vict. c. 102
  • 13. P a g e | 13 Liberals had to reconcile the ‘majority rules’ principle with the power of majority to be limited. The aspiration of accomplishing this in a manner consistent with democratic principles lead to the advent of a three-part liberal solution. 3.2.1. Principles of the Liberal Solution The first was the conception of the separation of powers. This was an adaptation of the trias politica principle of the Greeks which stipulated the distribution of powers between functionally differentiated agencies of government as the legislature, the executive and the judiciary. This formed as a system of check and balance so no single limb of the government held all the power. The next step of development was the implementation of periodic elections. This was a system that allowed for democratic decisions of the majority distributed over time. It pressures parliament to seek popular mandate and act on the voices of the majority that elected it. This expression of liberalism was the first step in the formation of the constitutional democracy which we know today – the checking of the current majority’s influence on decisions over the verdicts of majorities that precede it. The third part of the solution followed from liberalism’s commitment to the freedom and integrity of individuals. Liberals believe that even though a citizen shares a social contract with the community, they as individuals have rights which the state may not encroach upon. In order to properly implement a democratic model of governance, individuals must have rights such as freedom of expression, freedom to associate and organize and freedom from fear of reprisal. In liberal democracies, individuals should also maintain the private rights which concern not the state. These rights range from the practice of religion to the raising of children by their parents. Affirmations of these rights were declared in legal documents such as the British Bill of Rights (1689), the U.S. Declaration of Independence (1776) and Constitution (ratified 1788), the French Declaration of the Rights of Man and of the Citizen (1789). These documents accentuate the liberal perspective that
  • 14. P a g e | 14 freedom encompasses not only the occasional right to vote but fundamental rights of people to live their own private lives. It was the liberal insistence that “all men are created equal” that lead to a slow but steady expansion of the liberalism franchise which eventually led to the development of the liber secular model this dissertation is focused on. 3.3. Liberalism and Utilitarianism The next stage of development for Liberalism is the influence of utilitarianism. In the late eighteenth century, Jeremy Bentham20 and his student John Stuart Mill21 invoked a doctrine of utilitarianism- “it is the greatest happiness of the greatest number that is the measure of right and wrong.”22 This became a major element in the liberal conception of state policy objectives. This concept was inspired by the notion of a market economy. Utilitarians advocated a political system that would guarantee its citizens the maximum degree of individual freedom of choice and action consistent with efficient government and the preservation of social harmony. In John Stuart Mill’s On Liberty23 , he propounds on utilitarian grounds that the state may regulate individual behaviour only in cases where the interests of others would be perceptibly harmed. It is clear the influence utilitarianism has had on the philosophical foundations of political liberalism. Its classic advocacy of individual liberties have played an integral role in the modern liberalism model we understand today. In addition, the Utilitarian maxim of ensuring that the liberties afforded to the community is “consistent with efficient government and the preservation of social harmony” forms the basis of the understanding of the liberal secular model we know today. 20 Jeremy Bentham, (born February 15, 1748,London, England—died June 6, 1832, London), English philosopher,economist,and theoretical jurist,the earliestand chief expounder of utilitarianism. 21 John Stuart Mill,(born May 20, 1806,London, Eng.—died May 8, 1873, Avignon, France), English philosopher,economist,and exponent of Utilitarianism 22 Bentham, J., ‘A Fragment on Government.’ (London,1776), Preface (2nd para.). 23 Mill,John Stuart. ‘On Liberty’. (London: Longman, Roberts & Green, 1869);
  • 15. P a g e | 15 3.4. Liberalism and Equality 3.4.1. Classification of Liberalism There is a common misconception that liberalism places the interests of a part of society – the propertied classes, capitalists, the entrepreneurs- above the interests of other classes.24 This assertion is mistaken. Liberalism has always placed emphasis on the welfare of the whole, not that of any special group. Basil Mitchell identifies three kinds of liberalism (1) “A liberalism which bases itself on a conviction about the nature of man and of his destiny and which insists on the need of the individual for freedom to choose the way he shall live. It is a liberalism which has an explicit metaphysical context. (2) A liberalism which is sceptical of religious and metaphysical claims and which sees as the supreme values of human life the discovery of truth and the attainment of happiness. It regards the need for individual freedom partly as a self-evident truth, partly as a pre-condition of the discovery of truth and the attainment of happiness. (3) A liberalism which is sceptical not only of religious and metaphysical claims but of morality also, except in so far as it is a necessary condition of the survival of any society. It sets a high value on human beings and on the ideal projects men desire for themselves, although it does not regard these ideals as, in any sense, objective.”25 This summarises well the different variations of liberalism we know today. The first two kinds of liberalism presents a type of theory about the ‘good life’. The good life is an 24 Ludwig Von Mises,Liberalism (1 edn, The Foundation for Economic Education,Inc Irvington-on-Hudson,New York 10533 and Cobden Press 2002) 7 25 Basil Mitchell, ‘Law, Morality, and Religion in a Secular Society’ (Oxford: Oxford University Press,1967),p. 98.
  • 16. P a g e | 16 autonomous life, either because of some religious or metaphysical understanding, or because the good life involves ‘the discovery of the truth and the attainment of happiness’.26 The third kind of liberalism is different. While it lacks clarity on whether its scepticism denies the existence of moral truth claims, or it denies their relevance in public life, it is a liberalism that each human being chooses without any discrimination between ideals or projects.27 Joseph Raz claims that: “It (Liberalism) is identified by a series of political causes espoused by liberals over the centuries, by a variety of claims about the working of society and the economy, and by a cluster of ideas concerning the fundamental principles of political morality. … The specific contribution of the liberal tradition to political morality has been its insistence on the respect due to individual liberty.”28 It is in determining what this respect requires that have led to differing variations of the liberal political concept. For example, Dworkin propounds that the key value is ‘equality’ and the requirement that all citizens are treated as equals. He then asks what this requires in terms of political morality (the ethics, or ethical standards, of public or political life.): “What does it mean for the government to treat its citizens as equals? That is, I think, the same question as the question of what it means for the government to treat all its citizens as free, or as independent, or with equal dignity…It may be answered in two fundamentally different ways. 26 ibid. 27 Peter Griffith,'Christianity, liberalism and the separation of law and morals' [2001] 1(1) UCL Jurisprudence Review 28 Joseph Raz, ‘The Morality of Law’ (Oxford: Clarendon Press,1989),p. 1-2.
  • 17. P a g e | 17 (1) The first supposes that the government must be neutral on what might be called the question of the good life. (2) The second supposes that government cannot be neutral on that question, because it cannot treat its citizens as equal human beings without a theory of what human beings ought to be.”29 The first answer requires the neutrality of a government and if it prefers, in any decision or policy one conception of the good life over another, it cannot treat its citizens as equals. The second argues that governance requires an unbiased conception of what it is that makes life good, so as to treat each citizen in a way a good or wise person would expect. Dworkin is clear that in his view, ‘liberalism takes, as its constitutive political morality, the first conception of equality.’30 With this view, Dworkin agrees with John Rawls, who draws the distinction between views, such as those of Plato, Aristotle and the Christian tradition as represented by Augustine and Aquinus, which hold that there is only one conception of the good ‘to be recognised by all citizens who are reasonable and rational’31 , and the view that ‘there are many conflicting reasonable comprehensive doctrines with their conceptions of the good, each compatible with the full rationality of human persons, so far as that can be ascertained with the resources of a political conception of justice.’32 This aptly describes the complex relationship between liberalism and secularism. The development of liberalism lay the groundwork for the birth of secular theories. These theories, at least at their conception, were based on the benevolence of religion, reflecting magnanimous and optimistic assumptions that the nature of religion was cooperative. Where one can assume that religions are sensible and responsive, ready to dialogue and 29 Ronald Dworkin, ‘A Matter of Principle’ (Oxford: Clarendon Press,1986), p. 191. 30 Ibid. 31 John Rawls, ‘Political Liberalism’ (New York: Colombia University Press,1993),p.134. 32 ibid.,p. 135.
  • 18. P a g e | 18 compromise in the spirit of tolerance or irrelevance,33 pluralistic concessions are perhaps affordable and there are good practical reasons to legitimize such attitudes. It is by these assumptions that led to political and legal frameworks which lay emphasis on an accommodative approach in handling religious related disputes in public spheres in hopes of achieving equality – this approach will henceforth be referred to as “Equality-by- Accommodation”. But religion is not homogeneous and because attached to its doctrines is a necessity for ‘interpretation’ and ‘translation’, its social and worldviews will be ever-changing. As Toni Massaro aptly summarizes it: [S]ome religious faiths reject secular reason and pluralism outright as proper cultural baselines; and fidelity to most religious faiths implies conduct, not just belief or expression. Official support of religion therefore can mean reinforcement of conduct that undermines foundational liberal principles of critical inquiry, religious and ideological pluralism, and secular reason.34 It is when religion is socially or politically perceived as aggressive, that the accommodative attitude toward it has to change: “[T]he state’s duty [is] to protect its citizens against excessive demands of individual religious groups.”35 Adverting the risk of granting a public presence to practices which hold values contrary to a jurisdiction’s liberal constitution led to the evolution of a more “aggressive” liberal secularism which excludes religious influence from the public sphere – this will hence forth be referred to as “Neutrality-by-Exclusion”. 33 The Augustinian approach of Catholicismto secular power is not troublingto the worldly power. Reinhold Niebuhr and many others prefer reason to revelation when itcomes to moral truth. Reinhold Niebuhr, ‘Moral Man and Immoral Society: A Study of Ethics and Politics’ 28 (2001).These are easy cases for accommodation of religion. 34 Massaro,supra note17, at 943. 35 Karl-HeinzLadeur & Ino Augsberg, ‘The Myth of the Neutral State: The Relationship Between State and Religion in the Face of New Challenges’, 8 German Law Journal 143,148 (2007).
  • 19. P a g e | 19 4. Two Prongs of Liberalism – “Neutrality-by-Exclusion” vs “Equality-by- Accommodation” Religious conflicts in the seventeenth century manifested from a theological disagreement36 . Today, the contention stems from political disagreement. They are conflicts about whether religious people can bring their religion into the public sphere. In the United States of America, non-establishment is constitutionally protected and religious pluralism is at the foundation of the state.37 Whereas in Europe, there still lie many states with an established church and fairly homogeneous societies38 . However, European societies are becoming increasingly pluralistic, making conflicts in this region more visible. After discussing the historical context of liberalism, the dissertation will proceed to dissect further the two variations of the evolution of liberalism. The first view which holds that there is only one conception of the good ‘to be recognised by all citizens who are reasonable and rational’39 will be represented from an aggressive liberal secular perspective of “neutrality-by-exclusion” where the ‘one conception of good’ postulated earlier would be the benefit of the society as a whole as exemplified in the the legitimate aim of ‘living together’ followed by Laicite. The second, which is essentially a response to the former, will be represented from the a “equality-by-accommodation” perspective that embraces that ‘there are many conflicting reasonable comprehensive doctrines with their conceptions of the good, each compatible with the full rationality of human persons, so far as that can be ascertained with the resources of a political conception of justice.’40 Raz denies that a government can have no conception of what the good life is. The good life, is a life of autonomous choices from a 36 J. Ratzinger, ‘Truth and Tolerance: Christian Belief and World Religions’ (Ignatius Press,2004) 37 M. Nussbaum,‘Liberty of Conscience : In Defense of America’s Tradition of Religious Equality’ (Basic Books, 2008) 38 Andorra, Armenia, Denmark, UK Church of England (since Toleration Act 1689, c.13) and Church of Scotland (Church of Scotland Act 1921), Finland,Georgia,Greece, Iceland,Liechtenstein, Malta,Monaco, Norway 39 John Rawls, Political Liberalism (New York: Colombia University Press,1993),p. 134. 40 ibid.,p. 135.
  • 20. P a g e | 20 range of valuable options, and it is the role of the government to support and facilitate this multiplicity of views. This side will be discussed in light of the Canadian legal system which juxtaposed to the French, takes a more accommodative approach. By analysing this juxtaposition of approaches in the two legal systems, the dissertation evaluates the effectiveness of a liberal secular legal system which excludes religious influence from the public sphere in achieving equality. 4.1. “Neutrality-by-Exclusion”: Religion as a Threat Representatives of aggressive secularism like Andras Sajo41 are principally worried about religious movements that directly challenge secular arrangements. Andras Sanjo fears that religion may undermine ‘the legal arrangements that claim to be neutral and generally applicable to all people living in the national community’.42 The challenge is that religion forces secular legal systems to agree on compromises and concessions that imperil the integrity and coherence of secular laws.43 An example would be the growing encroachment on the freedom of expression for the sake of protecting religious sensibilities. Andras Sanjo argues that secularism is weak. The saga of Danish Jyllands-Posten case44 regarding religiously provocative cartoons shows that secularism is open to compromises and concessions that secular states should firmly refuse to make. He opines that religion does not have a place in the public sphere and thus welcomed the abolition of the blasphemy law in the UK.45 However, as far as the UK is concerned, the abolition does not make it aggressively secular. The abolition of their blasphemy law must be read in conjunction with the Racial and Religious Hatred Act 41 Sajo A., ‘Preliminaries to a Concept of Constitutional Secularism’,Int’l Const L, (2008),605 42 Ibid 43 Zucca,L., ‘Secular Europe : Law and Religion in the European Constitutional Landscape’, (1st edn, Oxford University Press,2012). 44 Henkel, Heiko (Fall 2010). "Fundamentally Danish? The Muhammad Cartoon Crisis as Transitional Drama" (PDF). Human Architecture: Journal of the Sociology of Self-knowledge. 2 VIII.Retrieved 26 April 2016. 45Sajo A., ‘Preliminaries to a Concept of Constitutional Secularism’,Int’l ConstL, (2008), 611 n.13 (referringto the Criminal Justiceand Immigration Act,2008,c.4, par 79 (Eng))
  • 21. P a g e | 21 2006, which still limits free speech with the right to be protected from hatred. In section 29J, after having defined what amounts to an act which stirs up religious hatred. The act states: “Nothing in this Part shall be read or given effect in a way which prohibits or restricts discussion, criticism or expressions of antipathy, dislike, ridicule, insult or abuse of particular religions or the beliefs or practices of their adherents, or of any other belief system or the beliefs or practices of its adherents, or proselytising or urging adherents of a different religion or belief system to cease practising their religion or belief system. “46 This act is an exemplification of what Lorenzo Zucca coins as presumption of priority.47 While the right to free speech sets the tone for the context we express ourselves, the fallacy lies in the assumption that our words have rights to absolute freedom. Acts like these carve out egregious exceptions which nonetheless limits our expression if it intends to provoke religious hatred. However, the criteria as to falls into this category of expression remains arbitrary and it is this arbitrariness that makes the legal system vulnerable to bias and inconsistency where it concerns the right to the freedom of expression. In Europe, free speech and secularism seem to play a more major role in society and comparative to the UK, there exists a presumption in favour of liberty. However, the courts seem to limit the freedom of expression as well, when confronted with the issue of blasphemy. In the seminal case of Otto Preminger Institute v Austria48 , the applicant institute tried to show a film that offended the Catholic religion and the religious feelings of the people of Tyrol, a region that consists of a large majority of Catholics in whose lives religion plays a very important role. The ECtHR was faced with the decision of whether the administrative sanction preventing the screening of the movie was in breach of Art.10 of the ECHR, or if it 46 Racial and Religious Hatred Act 2006,section 29J 47Zucca,L., ‘Secular Europe : Law and Religion in the European Constitutional Landscape’, (1st edn, Oxford University Press,2012). S2.2.1. 48 (13470/87) [1994] ECHR 26 (20 September 1994)
  • 22. P a g e | 22 was justified on grounds of protecting religious feelings. The court argued that the sanction was justified as the film risked provocation of a predominantly Christian population. Critics elucidate the fact that there should be no tension between free speech and the right to protection from being offended in a religious context, because strictly speaking, the latter is not a right.49 The point of contention lies in the fact that should such a right exist, it must apply to feelings in general, not only religious cases- the corollary of which would be a legal system that would undermine the essence of the right of free speech and expression. Where other forms of association claim equal protection, what warrants, as a matter of principle, special treatment when it comes to religious sensitivity? While it is an arduous task to answer this on political grounds, it is not difficult to justify it on a prudential one. The crux lies in the fact that because the nature of religiosity demands the deepest of attachments and identity, religious people are particularly susceptible to offence and are keen on responding to any perceived offence with any means, legal or otherwise. 4.1.1. “Aggressive Secularism“- Laicite The best illustration of aggressive secularism is the French notion of Laicite. It is characterized by two separate elements: legal Laïcité and ideological Laïcité. The former consists of ‘a very strict separation of church and state, against the backdrop of political conflict between the state and the Catholic Church that resulted in a law regulating very strictly the presence of religion in the public sphere (1905)’.50 The latter ‘claims to provide a value system common to all citizens by expelling into the private sphere.51 A clear exemplification of its application is the French legislature’s passing of a law prohibiting the wearing of most religious affiliated symbols, clothing, and garb in public schools.52 49 See Dworkin, R. ‘The Right to Ridicule’, NY Rev Books (23 March 2006),29 50 Id at xii. 51 Id at xiv. 52 Law No. 2004-228 of March 15, 2004, Journal Officiel dela R~publiqueFran~aise [J.O.][Official Gazette of France], March 17, 2004,p. 5190
  • 23. P a g e | 23 4.1.2. The State and Religious Exclusion from the Public Sphere:The Burqa Ban Controversy This dissertation will use the seminal case of S.A.S. v. France53 to explore the attitude and approach of the French with regards to neutrality and religion. In S.A.S. v. France54 , the ECHR upheld on July 1, 2014, the law to ban the wearing of clothing to conceal one’s face in public spheres. The applicant is a French national and resident born in 1990. As a devout Muslim, she wears the burqa and niqab for her religion, culture and personal convictions not out of coercion from anyone. She concedes that necessary circumstances like identity checks for security purposes justifies unveiling herself but otherwise, it is within her rights to express her religiosity and culture as there existed not any intention to annoy anyone else. The applicant essentially presented three submissions to the Court. Firstly, that the blanket ban on full-veil clothing in public spheres interferes with her rights under Articles 9 (right to freedom of thought, conscience and religion) and 10(freedom of expression) of the ECHR. It also trespasses on her right to respect for private life. Secondly, it was argued that the ban violated her right to freedom of association under Article 11(freedom of peaceful assembly and to freedom of association with others) alongside Article 14 (the prohibition of discrimination). Thirdly, that the ban should be viewed in light of Article 3 (the prohibition of torture or degrading treatment)55 . The freedom to privacy and freedom of religion or belief (Article 8 &9) was the dominant and recurrent theme in this case. The Court decided that, in adopting the law, the government had remained within the restrictions on Article 9(1) which are permitted by Article 9(2). 53 App. No. 43835/11,2014 Eur. Ct. H.R. 54 Ibid. 55 This will notbe delved into as itis not relevant to the dissertation hypothesis
  • 24. P a g e | 24 The Court accepted two of the legitimate aims submitted by the government. They were the grounds of the protection of public safety and protection of the rights and freedom of others. The Court expressed concerns about the severe consequences for women who wished to wear the veil and the extent of Islamophobia arguments hidden within the wider French debate regarding this law.56 However, the Court conceded that the State had grounds to find the law essential in upholding the requirements of “living together”57 in the French society. Concealing the face in public could be considered by the State as adversely affecting the “principle of interaction between individuals, which in its view is essential for the expression not only of pluralism, but also of tolerance and broadmindedness without which there is no democratic society”58 . The Court accepted the link between the aim to “the respect for the minimum requirements of life in society” and the protection of rights and freedom of others. Viewed from this prospective, the Court had to afford the State a wide margin of appreciation. This judgement reflects the legal applicability of the “neutrality-by- exclusion” model. Further reaffirming this fact, is that ECtHR is undeniably conservative when it comes to granting relief on issues involving Article 9 ECHR. For instance in Mann Singh v France59 , a Sikh man was denied his driving license as he had a turban on and identity regulation required a bareheaded photograph. In cases that involved removal of religious clothing in the educational environment like Dablab v Switzerland60 (teachers) or Leyla Şahin v Turkey61 , Kurtulmus v Turkey62 (students) for example, the Strasbourg Court has declared them either inadmissible or justified under Article 9(2) of the ECHR. The distinction of these cases lie in that there was a 56 Proportionality should only be considered when there was a general threat to public safety,the court rejected that there was a presence of this threat. 57 Id. atpp.142 58 Id. atpp 56 59App. No. 24479/07,2008 Eur. Ct. H.R 60App. No. 42393/98, 2001 Eur. Ct. H.R. 61App. No. 44774/98,2005 Eur. Ct. H.R. 62App. No. 65500/01,2006 Eur. Ct. H.R.
  • 25. P a g e | 25 conflict of religious freedom vis-à-vis duties attached to a social post. In this case, the choice to manifest personal religious convictions in the form of clothing should only be made during the applicant’s personal time. The only exception to this “harsh” judgement pattern is in the case of Ahmet Arslan and Other v Turkey63 , wherein Turkish nationals were convicted for wearing religious garments in public areas. The court granted relief in this case citing a breach of Article 9. The ECtHR was aware of the similarities between Ahmet Arslan and the present case, but were contented with the factual distinction that the present case involved a concealment of the face and accepted the “living together” principle. The S.A.S. v France case concluded the prevalent debate about the legality of the impugned law on banning face-concealing clothing. The Court has reasserted its margin of appreciation doctrine, showing that it was ready to interpret the Convention in the light of restrictive domestic approach to religious freedom. Although courts should not shy away from countering legislative majorities, it should nevertheless respect the sovereignty of the State, especially when said legislation has gathered unanimous support. Although unpopular, the Court’s endorsement of the French Republican approach to their restrictions on religious freedom elucidates an acceptance of their political and legal structure. The French jurisprudence follows a mix of Kant and Rousseau, positing that human beings are endowed with reason but do not always use it. It seeks not the just, but the good. Regardless of how the French Laïcité contradicts one’s personal conception of morality, the fact remains that the epistemology behind French jurisprudence follows that “Rights are not natural, but politically constructed”. 64 They can therefore be subject to 63App. No. 41135/98, 2010 Eur. Ct. H.R. 64 MilletFrancois-xavier ,'European Constitutional LawReview 2015 Case Comment' [2015] 1(1) When the European Court of Human Rights encounters the face: a case-note on the burqa ban in France <http://login.westlaw.co.uk/maf/wluk/app/document?&srguId=ia744d06500000151b361b42e1005b8a1&doc
  • 26. P a g e | 26 significant restrictions in the name of the common good of the political community. Although this approach does not attest to classical liberalism, it is an approach which is most compatible with the very ‘universalistic’ spirit of human rights underlying the Convention. A plethora of legal restrictions have been plagued by religious influence in ‘classically liberal’ societies. For example, restricting rights of homosexuals is undoubtedly a corollary of religious-politics and their biased conception of ‘evil’. In France however, same-sex marriage was recognized and passed regardless of the legislatures own personal religious beliefs. While concerns of islamophobia, gender discrimination or a general infringement of rights, are to an extent, relevant considerations; Laïcité, as are other political maxims, should be respected as a tool designed to manage diversity. Personal convictions, religious or otherwise, should be distinguished from the legal validity of a State’s political regime. In pragmatic terms, the choice of constitutionalism is a choice birthed from negotiation and allows for critical democratic discussions of the implementation of legislative law. However, as evidenced by Europe’s history with ecclesial law, when a religious organization comes into power, laws would not be of the people’s choice but made based solely of the religious leader’s interpretation of their doctrine. Even in the impossible case where all people voluntarily subscribe to a single religion, it cannot be said that policies from this religion are sovereign choices of the people. The choices will be that of (to a great extent pre-existing, taken for granted, non-negotiated) religion. The issue here is not that aggressive secular states like France do not comprehend religious arguments and their demands for special treatment. However, it draws a clear line between where the normative authority should lie. The fact is, the choice of secular constitutional arrangement is intellectually and practically available to all, but this is not true about an individual’s private religious beliefs. guId=I27D676D0799611E582588114149BF5E6&hitguId=I27D676D0799611E582588114149BF5E6&rank=6&sp os=6&epos=6&td=21&crumb-action=append&context=4&resolvein=true> accessed 20 April 2016
  • 27. P a g e | 27 4.1.3. Secularism vs. Secularization Andras Sajo supports a constitutional secularism that is aggressive (close to Laicite) capable of responding to religion and its presence in the public sphere. However, he bemoans the fact that we lack a strong normative theory of secularism to underpin our legal systems in order to eradicate such arbitrariness within legal systems. We learn that perhaps the problem of achieving equality lies with the perception of secularism itself. At this point, it is important to understand the difference in ‘secularization’ and ‘secularism’. Andras Sajo regards secularism as a social fact which stands for an ambiguous social reality, and that it is uncertain as a legal concept. Secularization on the other hand, is defined by Sajo as ‘a historical project still in the making’.65 For Sajo, it is about religion and its organizations conceding its power over various aspects of life in favour of the State.66 However, regarding secularization as a historical project is deemed problematic. Olivier Roy defines secularization as ‘a social phenomenon that requires no political implementation’.67 This holds the implication that secularization is not a power struggle between state and church, but a process of the gradual waning of religion in society. In this case, secularism and secularization may go hand in hand theoretically, with secularization as a process and the notion of secularism as a political project with a set of normative claims concerning the way in which the state deals with diversity. Practically, however, the resurgence of religion, raises doubts as to the direction of the political project and the facticity of the secularization social process. Zucca propounds that a comparative constitutional theory of secularism faces the problem that the notion of secularism is deeply intertwined with the local practices and histories in the west, as Charles Taylor has powerfully demonstrated.68 Andras Sajo is 65 Sajo A., ‘Prelimanaries to a Concept of Constitutional Secularism’,Int’l ConstL, (2008),609 66 Ibid. 67 Roy, O. Secularism Confronts Islam (Columbia University Press,2007),7 68 Taylor,C. A Secular Age (Harvard University Press,2007)
  • 28. P a g e | 28 similarly aware of the importance of social history and other contingencies in the formation of the Western understanding of secularism, explaining its weakness as that it is a ‘fuzzy constitutional concept’. According to Sajo, the project of secularization has never been coherently conceived and brought forward in the majority of legal and political systems. If secularism is weak and uncertain because of its local rootedness, secularization seems to be half hearted compromises by the church that are coincidently thought to be compatible with secularization itself. Andras Sajo argues that the project of secularisation is weak because it fails to display the intellectual consistency required to free the public sphere from religion. The crux of this argument lies in the perception of secularization, if one subscribes to Olivier Roy’s claim at secularization is not an intellectual project but an organic development of a society in response to the gradual waning of religion in people’s life, secularization would then mirror gradual development and thus, by definition, it seems to be an ideal that is never meant to be achieved. 4.1.4. Analysis: Secularism (Aggressive) is a Stronger Ideal than Perceived The author of this dissertation would like to point out certain doubts with regards to this perception of secularism. There is an inherent contradiction in ascertaining that the source of secularism’s uncertainty is its local rootedness and claiming that the process of secularization is an organic development of the waning of religion. There seems to be a perceived consensus that secularization is inconsistent because it is dependent on the cultural history of a jurisdiction. While it is undeniably true that domestic culture and history determines a society’s moral and political zeitgeist, what seems to have been overlooked is the universality of the shift from faith to reason. While our ancestors lived with faith “naively,” we can live with or without it “reflectively.” By contrast, we
  • 29. P a g e | 29 now live in a society in which, for the first time in history, “a purely self-sufficient humanism has come to be a widely available option.”69 Critics purport, that religious conflict is a symptom of mismanagement of pluralism within a secular state, not a threat to equality. However, this dissertation propounds that the accommodation of religion itself is the biggest obstacle to the ideal of equality. The cause for strong religious attachment lies in the fact that religion was at its roots manifested in regions that lack pluralism and diversity. It is precisely due to this fact that religious extremism seems to stem from jurisdictions, communities or even social groups which lack diversity. This explains the wane of religious influence over populations as time passes; globalisation has gradually imposed onto societies pluralism and diversity. The corollary of which is an environment which allows for intellect, reason and critical thinking to be nurtured, qualities of which are the building blocks for equality and peaceful co-existence. Over-sensitivity, extremism and blind adherence to religious doctrine – the causes of religious conflicts; stem from an absence of these qualities. The author hence propounds that secularisation should be viewed as an organic development or as Olivier Roy puts it, a ‘social phenomenon’ toward the ideal that is secularism- the intellectual ideal of achieving true equality void of obstacles caused by religion. By viewing it in such manner, we will overlook not the universal nature of the secularisation process and understand that while it may be organic, there still lies a need for positive actions (by governance) in order to achieve this ideal. Another flaw in the discussion earlier lies in Sajo’s definition of secularism as a social fact which stands for an ambiguous social reality - that it is uncertain as a legal concept. This dissertation concedes that it aptly depicts the state of secularism in most of today’s world. However, the ambiguity and uncertainties are only present when governance, influenced by political ambition or religious bias, do not take a strong enough stand in excluding all aspects 69 Ibid,p.18
  • 30. P a g e | 30 of society which have a propensity to incite discrimination and undermine equality – racism, sexism, religiosity etc. The theoretical validity and competence of secularism as a political and legal construct should not be dismissed with reasons of the likes of poor practical application. It is due to this that the dissertation propounds that “aggressive secularism” which practices “neutrality-by-exclusion”, if properly enforced, is the best approach toward achieving equality.
  • 31. P a g e | 31 4.2. “Equality-by-Accommodation” “Open Secularism” - Canada In order to fully comprehend secularism, it is essential to analyse the two variants ascribed to it, the first as discussed earlier, “aggressive secularism”, and the other “open secularism”. These competing visions of secularism were at the forefront of the highly publicized Bouchard-Taylor Commission70 that was constituted in Quebec, Canada to investigate the issue of accommodation practices in light of it being a pluralistic, democratic and egalitarian society. According to the commission, the four key principles constituting any model of secularism are: the moral equality of persons; freedom of conscience and religion; State neutrality towards religion; and the separation of Church and State. “Aggressive secularism” accords more importance to neutrality rather than to freedom of conscience and religion, attempting to relegate the religiosity to the private sphere, leaving the public sphere as “a neutral ground that stands outside religious controversy”.71 Contrastingly, “open secularism” is based on the protection of religion, even if this requires a relaxation of the principle of neutrality. In this model, state neutrality towards religion and the separation of Church and State are not seen as ends in themselves, but rather as the means to achieving the fundamental objectives of respect for religious and moral equality and freedom of conscience and religion.72 In “open secularism”, priority is given in favour of religious freedom and equality when there is any contradiction or tension between the various constituent facets of secularism. The crux lie in that this conception of 70 Named as such for the Commission‟s co-chairs,Gérard Bouchard and Charles Taylor.Its formal titleis the Consultation Commission on Accommodation Practices Related to Cultural Differences 71 Moon, R. “Introduction: Law and Religious Pluralism in Canada,” Law and Religious Pluralismin Canada,ed. Richard Moon (Vancouver: UBC Press,2008): 6 72 Jose Woehrlingand RosalieJukier, 'Religion and the Secular State in Canada' [2010] 1(1) Religion and The Secular State 185
  • 32. P a g e | 32 secularism is directed at state institutions rather than individuals and does not strive to neutralise religion as an identity marker in society. The “open secularism” stance is represented aptly by Chief Justice Dickson in the seminal case of R v Big M Drug Mart73 : “[a] truly free society is one which can accommodate a wide variety of beliefs, diversity of tastes and pursuits, customs and codes of conduct.”74 The Canadian courts apply this in their interpretation of the Canadian Charter of Rights and Freedoms75 . A more detailed review of Canada’s “open secularism” and a comparison to the French “aggressive secularism” will provide a more nuanced picture of the relationship between religion and the secular state. In doing so we will be able to competently evaluate the dissertation subject question of whether “[a] liberal secular legal system which excludes religious influence in the public sphere would adequately respond to the challenge of achieving equality”. 73 [1985] 1 S.C.R. 295 74 Id, at para.94,18 D.L.R. (4th) 321 75 Canadian Charter of Rights and Freedoms, Part I of the Constitution Act, 1982, being Schedule B to the Canada Act 1982 (U.K.), 1982,c.11 [hereinafter Charter]. See José Woehrling,“The Open Secularism Model of the Bouchard-Taylor Commission Report and the Decisions of the Supreme Court of Canada on Freedom of Religion and Religious Accommodation,” Religion,Culture and the State – Reflections on the Bouchard-Taylor Commission,eds. Howard Adelman and PierreAnctil (Toronto: University of Toronto Press,forthcoming in 2010).
  • 33. P a g e | 33 4.2.2. The State and Religious Autonomy: Analysis of the Kirpan Controversy In Canada, the state and religious communities operate in separate spheres and, in principle, it is not the role of the state, or secular courts, to intervene in their organization, nor to interfere in their autonomous governance.76 As Justice Iacobucci of the Supreme Court of Canada stated, “the State is in no position to be, nor should it become, the arbiter of religious dogma.”77 A clear juxtaposition in approaches to the attitude of the separation of Church and State can be seen in the reactions toward the French headscarf controversy78 . The French President Jacques Chirac, in response, characterized the law as necessary to maintain the religiously neutral nature of French schools and the tenet of secularism (laicite), or strict separation of church and state.79 However, the Supreme Court of Canada (S.C.C) responded in saying that it “"sent a strong message that Canada's public education institutions must embrace diversity and develop an educational culture respectful of the right to freedom of religion."80 In order to fully understand this ‘equality-by-accommodation’ approach taken by Canada, the case of Multani v. Commission scolaire Marguerite-Bourgeoys81 and the Canadian courts treatment of the kirpan wearing issue will be delved into. In Multani, the S.C.C. held that an orthodox Sikh student could wear a kirpan, a ceremonial dagger, to school given his strong religious beliefs. The Canadian courts have 76Woehrling,J. and Jukier, R. 'Religion and the Secular State in Canada' [2010] 1(1) Religion and The Secular State 190 77 Amselem, supra n. 41 at para 50. 78 Also known as “l'affaire dufoulard” 79 Press Release, U.S. Commission on Int'l Religious Freedom, “France: Proposed Bill May Violate Freedom of Religion” (Feb. 3, 2004), availableat http://www.uscirf.gov/mediaroom/press/2004/february/02032004_france.html; Walterick,supra note3, at 252; R. Murray Thomas, “Religion in Schools: Controversies in The World” 30-31 (2006) (statingthat purpose of laiciteis to ensure citizen loyalty to the state and the principles of freedom and neutrality).B 80 Terrance S. Carter and Anne-Marie Langan, “Supreme Court Gives Strong Endorsement to Freedom of Religion”, Church Law Bulletin No. 17 (Carters Prof I Corp., Orangeville,Ontario,Can.),Mar. 16, 2006,at 1, availableathttp://www.carters.ca/pub/bulletin/church/200 6/chchlb 17.pdf 81 [2006] 1 S.C.R. 256,
  • 34. P a g e | 34 addressed the issue of kirpan wearing in a number of lower court cases82 . In Tuli v. St. Albert Protestant Separate School District No. 683 , the plaintiff sought an injunction against the school board's policy that would have suspended or expelled him for wearing his kirpan to school.84 In a short decision, the court authorized the kirpan if blunted and held tightly within its sheath while on school property.85 The court found that allowing the kirpan would "provide those who are unfamiliar with the tenet of his faith an opportunity to be introduced to and to develop an understanding of another's culture and heritage."86 In Ontario Human Rights Comm. v. Peel Bd. of Education87 , found that a total ban of wearing kirpans in school was problematic as there is a lack of evidence suggesting that it posed a danger.88 The courts rejected the Peel Board of Education’s no-weapons policy with regards to categorizing the kirpan as a dangerous weapon rather than a religious symbol. The Board was unable to reach an acceptable compromise with affected members of the Sikh community such as wearing a replica or stitching the kirpan into its sheath to prevent its removal.89 In its judgement, the court considered that there were no reported incidents of school violence associated with kirpans but neglected the fact that at the time there had been incidents of violence associated with kirpans outside of the school setting including two stabbings.90 If the propensity of danger posed by the kirpan determines its categorization as a ‘dangerous weapon’, should there have been a distinction between acts of violence occurring in school or outside the premises? 82 Hothi v. R., [1985] 3 W.W.R. 256 (Can.), represents a prominent caseoutsideof the school context. In that case,the Manitoba Court of Queen's Bench upheld a provincial judge's decision to bar a Sikh defendant from wearing a kirpan in the courtroom duringhis trial.Seeid. 11. " 83 [1985] 8 C.H.R.R. D/3906 (Can.) 84 Id. ; BARNETT, supra note 5, at12. 85 Id. 7-8; BARNETT, supra note 5, at 12 86 Id. 4 87 [1991] 80 D.L.R. (4th) 475, 88 Id. at476. 89 Id. at476-77. 90 Id. at 477 See also Hamilton,supra note15, at 115 (citinga number of kirpan-related violentacts in Canada)
  • 35. P a g e | 35 Regardless, in light of the accommodative disposition of the Canadian courts, the court rejected a prohibition on the wearing of kirpans in schools by students and teachers upon consideration of the Ontario Human Rights Code 1981.91 The issue of kirpan in schools did not arise again in the courts until Multani reached the S.C.C. almost fifteen years later.92 In Multani, the S.C.C. unanimously agreed to set aside a court of appeal decision upholding a school board Council of Commissioners' prohibition on a student's kirpan wear.93 The issue began in late 2001 when Gurbaj Singh Multani's kirpan fell from his clothing when playing on a playground at Ecole Sainte-Catherine-Laboure, a French language school in Montreal. 94 Baptized Sikhs, like the Multanis, believe in five symbols of faith (the Five K's) including a comb (kangha), a pair of britches (kachha), a bracelet (karha), a head turban to cover uncut hair (keski), and a sword (kirpan).95 Sikhs view the kirpan as a religious symbol to be worn at all times rather than a weapon.96 But the school governing board eventually found that wearing a kirpan on school property would violate the school code of conduct's prohibition on weapons.97 Quebec's Attorney General intervened, issuing a statement emphasizing a zero-tolerance policy for "knives" in school, including kirpans.98 The S.C.C in deciding this case identified two issues. First, the prohibition from wearing his kirpan at school constituted an infringement of the student’s rights under the Quebec Charter or the Canadian Charter of Rights and Freedoms; Second, whether this 91 Id. at 480 92 WilliamJ.Smith, “Private Beliefs and Public Safety: The Supreme Court Strikes Down a Total Ban on the Kirpan in Schools as Unreasonable”, 16 EDUC. & L.J. 83, 83 (2006) (Can.) 93 Multani v. Commission scolaire Marguerite-Bourgeoys, [2006] 1 S.C.R. 256 (Can.); Canada Allows Sikh Students to Carry Daggers, Church and State, Apr. 1, 2006,at 21, availableat http://www.av.org/site/News2?page=NewsArticle&id8087&abbr--cs. 94 Stuart Laidlaw, “Accommodating Religions Challenges Secular Society; Supreme Court Shows Leadership with Recent Ruling Urges Schools to Teach About Their Students' Religions”, Toronto Star (Can.), Mar. 11,2006,at L09; WilliamJ. Smith, “BalancingSecurity and Human Rights: Quebec Schools Between Pastand Future”, 14 Edu. & L.J. 99, 111 (2004) (Can.) 95 Mann, supra note 33, at 61-62;Mcloed, supra note 33, at 32 96 Timeline: The Quebec Kirpan Case, supra note 31. 97 Multani,[2006] 1 S.C.R. 256, 4; Smith, supra note 31, at 111-12 98 Smith, supra note 28, at 93. Smith's analysis suggests theAttorney General's comments might have stemmed from broader public safety concerns such as 9/11 and terrorism.Smith, supra note 31,at 126-27
  • 36. P a g e | 36 infringement was justified under an exception in the Quebec Charter.99 The S.C.C held that there was an infringement on the student’s freedom of religion under S.2(a) of the Charter.100 The court acknowledged the presence of internal limits in S.2(a) which states that public safety could justify restricting a person’s freedom to practice religious beliefs, however, they declined to find such limitations applicable in this case.101 In R. v. Oakes102 , the S.C.C. set out the criteria used to evaluate restrictions on rights under the Charter. To qualify as a constitutional infringement of a Charter right, the government imposition must stem from an important objective, be rationally connected to an important objective, minimize impairment on an individual's rights, and embody proportionality between the objective sought and the imposition on the individual.103 In Multani, this test was used to consider the validity of the kirpan ban, however, the S.C.C focused more on accommodation and found a lack of evidence to support a total ban on kirpan wearing.104 In reaching its conclusion, the court noted "a total prohibition against wearing a kirpan to school undermines the value of this religious symbol and sends students the message that some religious practices do not merit the same protection as others."105 The case review of Multani elucidates Canada’s accommodative attitude toward freedom of religion, thus, the 1982 Canadian Charter of Rights and Freedoms remains a 99 Multani,[2006] 1 S.C.R. 256, at 13; Smith, supra note 31, at 115. An additional issueinvolved determining the proper standard of review for reviewing the school board's decision in an administrativecontext. See Smith, supra note 28, at 93-94.The majority of the court agreed to apply the stringent standard of review used in constitutional lawcases,buttwo minority opinions argued for an administrativelawstandard 100 Id at 33; 33; Bussey, supra note 31, at2. A more in-depth treatment of S. 2(a) follows. 101 Smith, supra note 28, at 96-97; Multani,[2006] 1 S.C.R. 256, 26 102 [1986] 1 S.C.R. 103 (Can.) 103 Id.; Luan-Vu N. Tran, “The Canadian Charter of Rights and Freedoms: Justification, Methods, and Limits of a Multicultural Interpretation”, 28 Column. HUM. RTs. L. REV. 33,58-59 (1996). 104 Smith, supra note 28, at 104. The minority opinion expresses concern with applyingaccommodation doctrines to public concerns.See id at 102.Because the court found a total ban unreasonable,itdid not have to consider the proportionality prongof the Oakes test. Id.at 105. 105 Multani,[2006] 1 S.C.R. 256, at 79.
  • 37. P a g e | 37 core document protecting individual liberties and expression.106 It serves to ensure priority is given to human dignity over the imposition of a standardized national ideology.107 Accordingly, under S.1, freedoms and rights articulated in the Charter are guaranteed "subject only to such reasonable limits prescribed by law as can be demonstrably justified in a free and democratic society."108 When considering religious freedom in the Canadian jurisdiction, the following provisions must be scrutinized. Firstly, s.2 of the Charter characterizes freedom of conscience and religion as a fundamental freedom held by "[e]veryone."109 This refers only to the expressly granted freedoms of religion, expression, press, and association, whereas the United States conception of fundamental freedoms is based on a larger collection of rights having either an express or implied basis in the U.S. Constitution.110 On the other hand, in France, freedom of religion is not fundamental, but rather, rejection of religious belief is seen as fundamental. Scholars theorize that France's recent acceptance of the weight of religious obligations may explain the country's difficulty in accepting religious difference.111 Second, s.15 requires that every individual be treated equally by and under the law "without discrimination based on race, national or ethnic origin, colour, [or] 106 Horwitz, supra note 51, at 3 (acknowledgingthat the preamble to the Canadian Charter serves as a reminder that "a citizen may have two sources of obligation");see also Tran,supra note 47, at 35 (callingthe Charter a "promisingdocument capableof mediating opposingvalues and interests"). 107 Tran, supra note 47, at 50, 54 108 Canadian Charter of Rights and Freedoms, Part I of the Constitutional Act,1982, being Schedule B to the Canada Act 1982, ch. 11 (U.K.), s.1. 109 Canadian Charter of Rights and Freedoms s. 2, PartI of the Constitution Act, 1982,being Schedule B to the Canada Act 1982, ch. 11 (U.K.) 110 Tran, supra note 47, at 42-43. 111 See Alain Garay,BlandineChelini-Pont,Emmanuel Tawil & Zarah Anseur, “The Permissible Scope of Legal Limitations on the Freedom of Religion or Belief in France”, 19 Emory International LawReview. 785, 819 (2005).
  • 38. P a g e | 38 religion . . . ."112 Finally, s. 27 calls for an interpretation of the Charter in a manner that preserves and enhances the "multicultural heritage of Canadians”.113 4.2.3. Analysis: Conceptual Flaws of the “Accommodative” Approach to Equality Canadian courts' treatment of religious freedom cases often begins with and is best exemplified by R. v. Big M Drug Mart114 ; it acknowledges that "in certain contexts minority communities suffer considerable disadvantages, even threats to their existence, if they are treated as identical to the majority group.”115 While the facticity of this statement cannot be argued, it simply accentuates the flaws in this accommodative approach towards achieving equality. There are essentially two conceptual flaws. First is the State’s concession to there never being a solution to the segregation of the population. While population segregation is organic especially due to the rise of pluralism in societies, it remains in essence the antithesis of equality amongst the population. One of the most crucial factors that cause social segregation, a casual factor of religious conflict, is the empowerment of contradictory religious dogma- which realistically can never be proportionate. If social segregation is at the root of the equality puzzle, can a legal approach that concedes to and even encourages this political and sociological flaw even be included in the equality conversation? Critics of secularism have a tendency to accentuate the main problem with the equality puzzle: neutrality is impossible. Secularists have no claim to neutrality because everyone has a set of presuppositions that guide their moral and ethical analyses. 112 Canadian Charter of Rights and Freedoms S. 15(1),Part I of the Constitution Act, 1982,being Schedule B to the Canada Act 1982, ch. 11 (U.K.). 113 Id. s.27 114 [1985] 1 S.C.R. 295 (Can.) 115 Tran, supra note 47, at 62.
  • 39. P a g e | 39 Contending for any position carries with it these presuppositions. It is unrealistic to claim that anyone interprets facts with zero bias where the mind is a ‘black slate’. The crux of this critique lies in the claim that there can be no neutral ground when taking a stance regarding the “human condition”116 and hence secularism no matter how aggressive cannot claim to be neutral. Any opinions and positions would inevitably be “normative, value-laden, metaphysical in character”117 For example, in the mid-1990s, the Human Embryo Research Panel established by the National Institutes of Health in the United States of America, where they discussed their deliberations on the federal funding of research on human embryos, declared, the “Panel weighed arguments for and against Federal funding of this research in light of the best available information and scientific knowledge and conducted its deliberations in terms that were independent of a particular religious or philosophical perspective.”118 However, Meilaender clarified the position of the opposing argument when he asserted, “We are not philosopher-kings who can adjudicate disputes between conflicting views without ourselves being parties to the argument.”119 The stance is premised on the assumption that we simply are unable to enter into bioethical discourse and leave our pre- suppositional framework at the door and hence it is argued that secularism cannot claim to be neutral. The author of this dissertation concedes that within every claim is premised upon some pre-suppositional framework. It is however, a fallacy to claim that because of the presence of a pre-suppositional framework, that secularism is not neutral. 116 The human condition is defined as "the characteristics,key events, and situations which composethe essentials of human existence, such as birth,growth, emotionality,aspiration,conflict,and mortality." 117 Meilaender,G. “Bioethics and the Character of Human Life,” The New Atlantis:1 (Spring 2003)http://www.thenewatlantis.com/archive/1/meilaender.htm (accessed April 25,2016). 118 Ibid. 119 Ibid.
  • 40. P a g e | 40 The distinguishing factor lies in the nature of these presuppositions. The argument for the exclusion of religion from the public sphere revolves around the fact that contradictory sets of stone-age religious dogma are asserting factual scientific claims with non-scientific evidence. These claims which directly affect the legal society, for example - the prohibition of stem cell research due to the Christian claim that it would constitute the killing of ‘souls’, are essentially premised upon faith based interpretative claims from a specific variant amongst the plethora of different non-evidential religious based claims of ‘absolute truth’. The standard of proof to any religious claim is hence, the blind faith in what is preached in an individual’s choice of a specific religious doctrine out of the variety of contradictory dogmas. In this case, it is undoubtedly true that pre-suppositional frameworks of this nature can never by neutral. On the contrary, in a liberal secular state which excludes religious influence, any factual claim would be held to a high standard of empirical evidence, intellectual debate and critical thinking – values of which can be said to be universal and hence foundationally neutral. If neutrality is purported to be an unachievable goal because any opinion would inevitably be “normative, value-laden, metaphysical in character”, then a liberal secular society which excludes religion is undoubtedly the best, most feasible method in achieving neutrality. A legal system void of religious influence in the public sphere would by nature, be one that strives to defeat these obstacles to neutrality. The insistence of an empirical, evidential and reasonable standard of proof to any claim, stems from scientific objectivity free from the arbitrariness of metaphysical claims. In fact, the secular position is one that is closest to being devoid of these pre- suppositions. It is by nature not a position that lays claim to any absolute truth, but a mechanism run by universalistic intellectual criteria to facilitate the democratic reaching of consensuses free from unsubstantiated claims to absolute truths.
  • 41. P a g e | 41 Secondly, there seems to be an assumption that the only alternative to accommodating the religious minority group is an adherence to the belief of the majority. This fallacy, which presupposes a necessity of bias, is again one that fundamentally contradicts the notion of neutrality and equality. An insistence on ‘choosing a side’, by definition implies that a group has been favoured over another. It is due to this that the author of this dissertation supports a more aggressive ‘neutrality-by-exclusion’ secularism. It is also integral to note that there should be no claim that choosing secularism over religiosity is in itself a bias. The contradictory nature of religious dogma relative to each other makes achieving equality by means of accommodation not only conceptually flawed but realistically unattainable. The “equality-by-accommodation” position is one that embraces the assertion of “religious absolute truths”- the embracing of which, relative to other religions, is a manifestation of bias. On the contrary, a secular position which excludes any form of religious influence from the public, is one that strives to achieve equality without any assertions of any “absolute truths”. It is one with a raised standard to proof (empirical, evidential and scientific) when it comes to the evaluation of any factual claims. These two positions are not comparable because the former embraces the assertions of “absolute truths” which are inherently biased, while the former makes no assertions but instead, as previously discussed, implements higher standards of proof as a system of check and balance – a neutral position.
  • 42. P a g e | 42 5. Conclusion Throughout history the place and role of religion in the public sphere has been generally addressed through a binary model, scrutinizing the juxtaposition between secular and religious. In order to achieve equality and be truly neutral towards different religious (and non-religious) conceptions of life, the fact is, the public sphere cannot but be secular. Public decisions affecting economics, politics and law must be reason-based not faith-based in order to sustain public order as ordered liberty. Constitutionalism exists only where political powers do not ground their public affecting decisions on transcendental concerns. Andras Sajo aptly describes this by the example, “People are buried in cemeteries not because it facilitates resurrection but for public health reasons.”120 Aggressive secularism as an institutional arrangement and legal framework is a shield for reason-based polity121 against a social “(dis)order” based on dictates of religious doctrine. This makes it the best response to the challenge of achieving equality. It is crucial to understand that the purpose of the aggressive secular position propounded is not to just exclude religion; exclusion is but the means to achieving equality. Secularism is about a form of communication accessible to all (believers or non-believers), relying on empirical, substantiated critical discussion. It provides a universal standard for judging what deserves normative authority. It is illogical for one to claim that the sensitivity that follows the nature of religious attachment deserves any special treatment. Religious sensitivity is, in a way, related to identity. It does not command specific legal respect.122 The opposite would be an abuse of the exceptionalism of free exercise. The insistence of a reason-based political and legal society dictates that legal choices be based on secular public reasons – which are reasons accessible to all, 120 Andras,S. ‘Constitutionalism and Secularism: The Need For Reason ' [2009] 30(6) Cardozo Law Review 2401 121 an organized political community livingunder a singlesystem of government of which policies are determined by reason 122 András Sajó, “The crisis that was not there: Notes on a reply”, IntJ Constitutional Law(2009) 7 (3): 515-528 firstpublished onlineJune15, 2009 doi:10.1093/icon/mop014
  • 43. P a g e | 43 regardless of their religious belief. Religiously grounded reasons require an element of “interpretation”. Without secularism’s insistence on reason-giving, faith-based divine command accessible only to the believer will potentially become acceptable and even mandatory. Allowing religious influence in law and the public sphere, undermines on a fundamental level, the principles of constitutionalism and most relevantly and particularly – equality. If popular sovereignty presupposes that all members of the community have reasoning capacity that enables them to participate in political decision-making, ‘blind’ faith by definition undermines it conceptually. Critics of secularism stand by ‘the inability of the secular state to cope with the fact of pluralism’. However, as Sajo states, ‘it is not the secular or non-secular nature of the state that disables its capacity to deal with pluralism’. If ideals, religious or otherwise, threaten the cohesion of our polities, then it becomes the concern of the shared epistemological foundations and the possibility of communication within the constitutional political and legal community. It is only with epistemological unity that enables peaceful and equal co-existence. The fact is, the aggressive secular position does not propose exclusion of religion simply in order to cope with disagreement. The issue is that religious groups, especially the major organized ones, are not concerned with disagreement. They seek exclusive imposition of their divinely commanded positions. That is why the author consistently accentuates the fact that an aggressive secular system is the best way to achieve equality. There will no doubt be objections by the religious groups and secularism offers space for disagreement, but as the dissertation has clearly established, religious-based demands should be strictly be confined to the private sphere. Equality, unlike what many critics purport, is not an unachievable ideal. But it is one that requires the competence of governing body to properly enforce the “neutrality-by- exclusion” concept and the endurance to manage any collateral conflict. It is one that
  • 44. P a g e | 44 requires time. Time for the population to gradually understand that the relegation of religion into the private sphere is for the betterment of the legal, political and social dynamics of the society. The dissertation has established that accommodating to religious demands is conceptually incompatible with any notion of equality, it has showed the legal validity and dismissed the misconceptions regarding the uncertainty and ambiguity of the aggressive secular position. But most importantly, this dissertation has established that a secular system that excludes religious influence from the public sphere is the best way to achieve equality. [Word Count:10,486]
  • 45. P a g e | 45 Bibliography Table of Cases  Ahmet Arslan and Other v. Turkey, Application No. 41135/98, 2010 Eur. Ct. H.  Braunfeld v. Brown, 366 U.S. 599 (1961)  Dahlab v. Switzerland, Application No. 42393/98, 2001 Eur. Ct. H.R.  Hothi v. R., [1985] 3 W.W.R. 256 (Can.)  Kurtulmus v Turkey, Application No. 65500/01, 2006 Eur. Ct. H.R.  Mann Singh v. France (dec.), App. No. 24479/07, 2008 Eur. Ct. H.R  Multani v. Commission scolaire Marguerite-Bourgeoys [2006] 1 S.C.R. 256,  Otto Preminger Institute v Austria (13470/87) [1994] ECHR 26 (20 September 1994)  Ontario Human Rights Comm. v. Peel Bd. of Education [1991] 80 D.L.R. (4th) 475  Reynolds v. United States, 98 U.S. (8 Otto.) 145 (1878)  R. v. Oakes [1986] 1 S.C.R. 103 (Can.)  R. v. Big M Drug Mart [1985] 1 S.C.R. 295 (Can.)  R v Big M Drug Mart [1985] 1 S.C.R. 295  S.A.S. v. France [GC], App. No. 43835/11, 2014 Eur. Ct. H.R.  Sahin v. Turkey, Application No. 44774/98, 2005 Eur. Ct. H.R.  Tuli v. St. Albert Protestant Separate School District No. 6 [1985] 8 C.H.R.R. D/3906 (Can.) Table of Legislation  Church of Scotland Act 1921  Canadian Charter of Rights and Freedoms, Part I of the Constitution Act, 1982, being Schedule B to the Canada Act 1982 (U.K.), 1982  Criminal Justice and Immigration Act 2008  European Convention of Human Rights Art.3,8,9,10,14
  • 46. P a g e | 46  Law No. 2004-228 of March 15, 2004, Journal Officiel de la R~publique Fran~aise [J.O.][Official Gazette of France], March 17, 2004,  Racial and Religious Hatred Act 2006  Toleration Act 1689 Secondary Sources Books  Adams J., ‘A Defence of the Constitutions of Government of the United States of America’ (3rd edn, The Lawbook Exchange Ltd, 1787)  Dworkin R., ‘A Matter of Principle’ (Oxford: Clarendon Press, 1986),  Dworkin R., ‘The Right to Ridicule’, NY Rev Books (23 March 2006)  Kymlicka W., ‘Multicultural Citizenship: A Liberal Theory of Minority Rights’, (Oxford: Oxford University Press 1995), 1995.  Locke J., ‘A Letter Concerning Toleration and Other Writings'. (Indianapolis: Liberty Fund, 2012).  Ludwig Von Mises, ‘Liberalism’ (1st edn, The Foundation for Economic Education, Inc Irvington-on-Hudson, New York 10533 and Cobden Press 2002)  Roy O., Secularism Confronts Islam (Columbia University Press, 2007)  Raz J., ‘The Morality of Law’ (Oxford: Clarendon Press, 1989)  Rawls J., ‘Political Liberalism’ (New York: Colombia University Press, 1993)  Sajo A., ‘Preliminaries to a Concept of Constitutional Secularism’, Int’l Const L, (2008),  Zucca L., ‘A Secular Europe: Law and Religion in the European Constitutional Landscape’ (1st edition, Oxford University Press 2012)
  • 47. P a g e | 47 Journals  Basil Mitchell, ‘Law, Morality, and Religion in a Secular Society’ (Oxford: Oxford University Press, 1967  Bader V., “Secularism or Democracy? Associational Governance of Religious Diversity”(2007)  Bentham J., ‘A Fragment on Government.’ (London, 1776),  Dieter Grimm, 'General Laws and Religious Norms' [2009] 30(6) Conflicts Between General Laws and Religious Norms  Ferrari S., 'Law and religion in a secular world: a European perspective' [2012] 1(1) Ecclesiastical Law Journal  Galanter M., ‘Religious Freedoms in the United States: A Turning Point’ [1966] Wisconsin Law Review 2  Garay A., Chelini-Pont B., Tawil E. & Anseur Z., “The Permissible Scope of Legal Limitations on the Freedom of Religion or Belief in France”, 19 Emory International Law Review. 785, 819 (2005).  Hosen N. and Mohr R., Law and Religion in Public Life (London, 2011).  Habermas J., ‘Notes on a Post-secular Society’ (2008)  Henkel H., (Fall 2010). "Fundamentally Danish? The Muhammad Cartoon Crisis as Transitional Drama" (PDF). Human Architecture: Journal of the Sociology of Self- knowledge. 2 VIII. Retrieved 26 April 2016.  Karl-Heinz Ladeur & Ino Augsberg, ‘The Myth of the Neutral State: The Relationship Between State and Religion in the Face of New Challenges’, (8 German Law Journal 2007)  Laidlaw S., “Accommodating Religions Challenges Secular Society; Supreme Court Shows Leadership with Recent Ruling Urges Schools to Teach About Their Students' Religions”, Toronto Star (Can.), Mar. 11,2006
  • 48. P a g e | 48  Luan-Vu N. Tran, “The Canadian Charter of Rights and Freedoms: Justification, Methods, and Limits of a Multicultural Interpretation”, 28 Column. HUM. RTs. L. REV. 33, 58-59 (1996).  Meilaender G., “Bioethics and the Character of Human Life,” The New Atlantis: 1 (Spring 2003)http://www.thenewatlantis.com/archive/1/meilaender.htm (accessed April 25, 2016).  Mill, John Stuart. ‘On Liberty’. (London: Longman, Roberts & Green, 1869);  Millet Francois-xavier , 'European Constitutional Law Review 2015 Case Comment' [2015] 1(1) When the European Court of Human Rights encounters the face: a case- note on the burqa ban in France <http://login.westlaw.co.uk/maf/wluk/app/document?&srguId=ia744d06500000151b3 61b42e1005b8a1&docguId=I27D676D0799611E582588114149BF5E6&hitguId=I27 D676D0799611E582588114149BF5E6&rank=6&spos=6&epos=6&td=21&crumb- action=append&context=4&resolvein=true> accessed 20 April 2016  Moon R., “Introduction: Law and Religious Pluralism in Canada,” Law and Religious Pluralism in Canada, ed. Richard Moon (Vancouver: UBC Press, 2008):  Murray Thomas R., “Religion in Schools: Controversies in The World” 30-31 (2006)  Nussbaum M., ‘Liberty of Conscience : In Defense of America’s Tradition of Religious Equality’ (Basic Books, 2008)  Niklas Luhmann, ‘Observations on Modernity’ (1st edn, Stanford University Press 1998)  Press Release, U.S. Commission on Int'l Religious Freedom, “France: Proposed Bill May Violate Freedom of Religion” (Feb. 3, 2004), available at http://www.uscirf.gov/mediaroom/press/2004/february/02032004_france.html;  Peter Griffith, 'Christianity, liberalism and the separation of law and morals' [2001] 1(1) UCL Jurisprudence Review
  • 49. P a g e | 49  Raphael Cohen-Almagor, “Ethnocultural Minorities in Liberal Democracies”, in: R. Cohen-Almagor (ed.), Challenges to Democracy: Essays in Honour and Memory of Isaiah Berlin, London: Ashgate Publishing Ltd., 2000  Reinhold Niebuhr, ‘Moral Man and Immoral Society: A Study of Ethics and Politics’ 28 (2001)  Ratzinger J., Truth and Tolerance: Christian Belief and World Religions (Ignatius Press, 2004)  Sajó A, ‘Constitutionalism and Secularism: The Need For Reason ' [2009] 30(6) Cardozo Law Review 2401  Sajó A., “The crisis that was not there: Notes on a reply”, Int J Constitutional Law (2009) 7 (3): 515-528 first published online June 15, 2009 doi:10.1093/icon/mop014  Terrance S. Carter and Anne-Marie Langan, “Supreme Court Gives Strong Endorsement to Freedom of Religion”, Church Law Bulletin No. 17 (Carters Prof I Corp., Orangeville, Ontario, Can.), Mar. 16, 2006, at 1, available at http://www.carters.ca/pub/bulletin/church/200 6/chchlb 17.pdf  Tourkochoriti I., ‘The Burqa Ban: Divergent Approaches to Freedom of Religion in France and in the USA’ 20(3) William and Mary Bill of Rights Journal (2012)  Taylor C., A Secular Age (Harvard University Press,2007)  Woehrling J., “The Open Secularism Model of the Bouchard-Taylor Commission Report and the Decisions of the Supreme Court of Canada on Freedom of Religion and Religious Accommodation,” Religion, Culture and the State – Reflections on the Bouchard-Taylor Commission, eds. Howard Adelman and Pierre Anctil (Toronto: University of Toronto Press, forthcoming in 2010).  Woehrling J. and Jukier R., 'Religion and the Secular State in Canada' [2010] 1(1) Religion and the Secular State
  • 50. P a g e | 50  William J. Smith, “Private Beliefs and Public Safety: The Supreme Court Strikes Down a Total Ban on the Kirpan in Schools as Unreasonable”, 16 EDUC. & L.J. 83, 83 (2006) (Can.)  William J. Smith, “Balancing Security and Human Rights: Quebec Schools Between Past and Future”, 14 Edu. & L.J. 99, 111 (2004) (Can.)